Этого треда уже нет.
Это копия, сохраненная 29 января 2021 года.

Скачать тред: только с превью, с превью и прикрепленными файлами.
Второй вариант может долго скачиваться. Файлы будут только в живых или недавно утонувших тредах. Подробнее

Если вам полезен архив М.Двача, пожертвуйте на оплату сервера.
Тред тупых вопросов №133 Leavitt edition 585265 В конец треда | Веб
Тред вопросов о жизни, Вселенной и всём таком.

Спрашиваем то, за что в других местах выдают путёвку в биореактор. Здесь анонимные учёные мирового уровня критически рассмотрят любые гениальные идеи и нарисованные в Paint схемы.

Предыдущий тут: >>582065 (OP)
https://2ch.hk/spc/res/582065.html (М)

Q: Можно быстрее?
A: Можно упасть в пузырь Альбукерке, NASA уже почти надула его.

Q: Я начитался охуительных историй про уфологию, че делать, нам жопа?
A: Да, тебе жопа, можешь сгонять в зогач или куда оттуда пошлют.

Q: Что будет с человеком в вакууме без скафандра / если он упадет на черную дыру / попробует ступить на поверхность газового гиганта/солнца?
A: Он умрёт.

Q: Почему бы не привязать ракету к воздушному шару или стартовать с горы?
A: Космос - это не как высоко, а как быстро, большая часть энергии ракеты уходит на разгон вбок.
Подробнее тут https://what-if.xkcd.com/58/ (английский) https://chtoes.li/orbital-speed/ (перевод)
2 585267
>>5265 (OP)
Ебанный стыд...
Во-первых, Алькубьерре.
Во-вторых, не упасть, а создавать вокруг корабля изнутри (иначе кина не будет).
В-третьих, НАСА искривляет пространство на десятимиллионную часть, контролируя
это сверхточными интерферометрами, до самого варп-привода здесь - как до
Антарктиды раком.
159298325425500.jpg92 Кб, 688x450
3 585281
RozalijaZalkind.jpg67 Кб, 350x440
4 585291
5 585292
>>5291
Ну и нахуя оно тут?
6 585293
>>5265 (OP)
Почему Скотт Мэнли такой охуенный?
Есть ли аналогичные по информативности каналы?
Я как запускаю видео Скотта - обнаруживаю что 10 минут прошли как одна, никогда не скучно. Каждодневный астронавт как-то уныло пургу гонет и канал будто на его личности (и муска) акцентирован, а не на космических фактах.
Любопытный Дроид относительно нормальный вспомнился, и все, больше никого не упомню.
7 585299
>>5267
Если на корабле им. Аль Букерке перевезти флешку с информацией, будет ли это считаться за передачу информации быстрее скорости света?
8 585303
>>5299
Что за фиксация на флешках? Даже слово ХУЙ, написанное на борту корабля будет являться информацией. Как впрочем и отсутствие этого слова.
9 585355
>>5293
Он был бы охуенен, если бы своим ебалом меньше светил на весь экран. Я бы даже подписался, рассказывает неплохо.
10 585357
>>5355
Он не светит, когда есть чего показать.
Последнее видео про докинг адаптер - 95% хронометража - хроники со стыковочными узлами.
11 585457
Какие есть крутые двигатели, которые человечество теоретически сможет создать в этом веке? Хочется чтобы было как в сериале "Экспанисия" - летишь к любой точке в солнечной системе с постоянным ускорением в 10-15 м/с^2 на огромном корабле, который не состоит на 90% из топлива.
На ум приходит только двигатель на ании-материи, но чет я сомневаюсь, что в этом веке люди научатся ее добывать и хранить в промышленых масштабах.
12 585459
>>5457
Взрыволет на нюках.
13 585461
>>5457
Такая термоядерная хуета вполне реальна
14 585464
>>5459
Так ускорение будет импульсным. Не думаю, что будет комфортно неделю лететь в невесомости, когда тебя каждые несколько секунд колбасит об нос корабля.
Хотя, эта идея наиболее реалистична. Мб если пускать очень часто очень маленькие бомбы, то и норм будет.
15 585469
Одно из решений парадокса Ферми в том, если не ошибаюсь, что на определенном этапе развития разумные цивилизации закономерно замыкаются в себе, т.е. утрачивают интерес к видимой внешней вселенной. Что интересного почитать на эту тему?
16 585514
>>5469
Историю за последние 50 лет, вместо колонизации Луны и Марса развиваем интернеты и прочие сферы развлечений.
15930369180211791545018318202595.jpg40 Кб, 800x499
17 585523
18 585524
>>5469
айзек азимов конец вечности
19 585533
>>5469
Парадокс Ферми апеллирует к идиотскому тезису, что инопланетяне с какого-то хрена будут общаться в радиодиапазоне и срать сигналами во все стороны в рамках своей программы SETI. Даже гипотетический межзвёздный лазерный модем обнаружить с Земли практически невозможно, а уж если инопланетяне используют сверхсветовой канал связи - обнаружить их нашими методами невозможно вообще никак.
20 585536
>>5533

>Даже гипотетический межзвёздный лазерный модем обнаружить с Земли практически невозможно


В фокусе гравилинзы можно, но это надо на каждую изучаемую звезду отдельный телескоп в пояс Койпера запускать.
21 585537
>>5514
Так вот я к тому что может как раз это и не плохо, а хорошо. Если допустить, что все пошли этим путем, так может и нам надо туда мм? Может мы просто не там ищем путь к звездам? Может надо вкладываться в ветку психофармакологии, нейрофизиологии, медитации, религии в конце концов?
22 585538
>>5536
Они могут еще замаскировать свой канал рандомными шумами, так что без ключа ты в шумах сигнал не найдёшь, будет просто как нытье какой-то переменной звезды.
23 585539
>>5537
Вкладывайся, дружище. Выкидывй свой компьютер и вперёд, езжай в Камбоджу на границу с Лаосом за просветлением.
24 585540
>>5539
Ты можешь мыслить без стереотипов? Заметь, раздел называется "Космос и астрономия", а не "Наука о космосе", с этим у нас все нормально.
Я что имею в виду, собственно антропный принцип. Вселенная такая какой мы ее видим, потому что можем осознавать ее только посредством нашего разума. Следовательно, это вселенная человека до тех пор, пока мы остаемся существами с разумом человека. Тот же пример про моль, поедающую книгу - для моли реальности текста не существует, а для человека реальность текста имеет большую ценность, чем реальность бумаги. Можно ли по аналогии представить себе такую же реальность, которая будет текстом по отношению к человеческой вселенной? Вот в чем мой вопрос.
Я вписал религию в список, потому что это именно то на что пытаются ответить все религии. "Будьте как боги, и войдете в царствие небесное". Человеческий разум 2 миллиона лет развивался вообще-то, откуда это 150-летнее высокомерие "все мы изобрели радиотелескоп, все хуйня давай по новой". Не пойдет!
25 585541
>>5540
Всё это словоблудие про психонейромидатцию лишь троянский конь для пропуска за барьеры скетицизма науки, чтобы потом из него как по плану выскакивал какой-нибудь пастух семитской религии, как ответ на все словоблудские набросы. Плавали знаем, нахуй проследуй, Ленин-гриб.
26 585556
>>5303
Тащемта уже само наличие этого корабля в точке XYZ будет информацией.
27 585557
>>5457
Никакие. Предпосылок к этому ровно 0, только манясхемы, которые изменятся 500 раз даже если кто-то займётся. А ванговать за пределами хоть каких-то предпосылок - все равно что рулетку крутить
28 585559
>>5469
Ничего. Все рассуждения на эту тему - фикшен. Вымысел, основанный на куче допущений, одно из которых назвал анон >>5533 . Разве что может быть Лема, который настолько абстрактен и нематериален, что при рассуждении об этом парадоксе его даже не касается (что само по себе парадокс). Может быть Игана ещё, там хоть вымысел, но очень высокоуровневый, а не чистое фэнтези.
29 585561
>>5537

>психофармакологии, нейрофизиологии, медитации, религии в конце концов?


И как это поможет телепортации к звездам? Может это научит видеть рентген?
image.png33 Кб, 350x350
30 585562
>>5561

>научит видеть рентген?


Я и так его вижу, он весьма заметный в центре рентген-кабинета стоит.
31 585585
>>5559

>Лема, который настолько абстрактен и нематериален


>>5559

>Лема, который настолько абстрактен и нематериален


Чо? Ты чо? Уж реальней Лема вообще ничего быть не может, он материалистичен до абсолюта.
32 585605
>>5265 (OP)
ребятки, я к вам с очередной платиной пришел..
вот типа нельзя лететь далеко из-за повышенной радиации. почему нельзя создать защиту из магнитного поля, развесив по корпусу электромагниты и т.д., для создания защитного поля? чисто умозрительно, теоретически, без учета технических проблем?
unnamed (1).jpg103 Кб, 502x351
33 585606
>>5561

>как это поможет телепортации к звездам?


Телепортация тушки и не нужна. Между телепортацией и просто межзвездным перелетом нет концептуальной разницы для данного рассуждения, гостей-то мы и так и так не наблюдаем.
О чем речь, допустим, микрожитель микросхемы изобрел телескоп. Он смотрит на окружающий мир, что он видит? Пластик, железяки, пыль, присохшую кошачью шерсть, дохлых тараканов, крепления к корпусу и т.д. - красная стрелка. Расстояния велики, достигнуть даже ближайших наблюдаемых структур в разумное время он не может. А надо разгадать свое назначение и научиться принимать и отдавать сигналы (синяя стрелка). Тогда коммуникация мгновенная, и вся система к тому же приобретает смысл.
Если что я не пытаюсь кого то в чем то убедить этой дичью, просто думаю насколько иным в принципе может быть подход. Пространственно-временной континуум тоже ведь неочевидная штука, а вон как оно вышло в итоге.
34 585607
Ты как гамма-излучение и нейтроны примагничивать собрался?
35 585608
36 585609
>>5605
Во-первых, магнитное поле влияет не на любую радиацию, а только на ту, что состоит из заряженных частиц (гамма и нейтроны мимо, например). Во-вторых, подозреваю, при напряженности поля, достаточной для защиты от радиации на дистанции в пару метров, тебе просто поплохеет, пушо на тушку это тоже будет оказывать влияние.
37 585611
>>5607
они будут ударятся о заряженные частицы в поле и остановяться
38 585615
>>5611
Частиц там должно быть как минимум как в метровой толщине свинцовой пластине.
39 585616
>>5615
а если они будут быстро летать то успеют зацепить все нейтроны же
40 585630
>>5616
Как ты себе это представляешь? Нейтроны же не пешком ходят, даже тепловые нейтроны тоже летят на скорости в несколько километров в секунду. И это мы еще о гамме не начали говорить.

У тебя никак не получится сделать защиту, эквивалентную метру свинца, без запихивания в щит количества вещества, сравнимого с этим самым метром свинца.
41 585659
Я так понял у кого-то новая вайфа спейсача.

Вопрос мой в следующем. На сколько можно доверять этому http://www.psrd.hawaii.edu/Jan05/MarsRecently.html
?
42 585665
>>5659

>Я так понял у кого-то новая вайфа спейсача.


Что это значит?
43 585671
Смотрите.
Берём запутанную пару. Когда мы коллапсируем одну то одновременно коллапсирует вторая. Если мы измерим вторую, то мы сколлапсируем и первую. Возьмём массив запутанных пар. Отправляем каждый день на далёкую звезду массивы с запутанными частицами. Они идут постепенно и равномерно, не обязательно на релятивистских скоростях. В один момент эта линия из массивов достигает звезды. Через определённое время, на земле решили передать информацию к звезде. Мы открываем запасники и коллапсируем пару, которая должна была прийти завтра. А в это время у далёкой звезды помнят инструкцию оставленную столетиями ранее и каждый раз при приходе посылки они измеряли пары на предмет запутанности. Каждый раз она разрушалась. Но в обозначенный день они проверили и обнаружили, что пара уже разрушена или не разрушена. Что будет обозначать информацию в 1 бит переданную на расстояния многие световые года за день.

Где ошибка?
44 585673
45 585679
>>5673
Это херня. Была другая (забыл как звали), дважды за полгода в шапке ТТВ красовалась.
46 585681
>>5671

>каждый раз при приходе посылки они измеряли пары на предмет запутанности


>Но в обозначенный день они проверили и обнаружили, что пара уже разрушена или не разрушена



Такого измерения не существует, запутанность это не какое-то отдельное свойство, а просто тот факт, что некоторые свойства двух частиц всегда коррелируют между собой. При любом измерении запутанность необратимо разрушается и обе частицы принимают противоположное состояние. Самый простой пример — спин, если у одной частицы его направление было измерено в одну сторону, то у другой всегда будет точно в противоположную.

Ты измеряешь спин частицы и он, например, направлен вверх. Это вообще ничего не говорит тебе о том, была ли до этого измерена парная ему частица, и если да, то когда. Всё, что запутанность тебе говорит, это то, что у парной частицы спин теперь направлен вниз, больше из неё нельзя извлечь ни единого бита информации. Повлиять на направление спина, не разрушив запутанность, тоже нельзя.

Короче, квантовая телепортация это просто способ получить полностью случайную, но 100% скоррелированную информацию в двух местах одновременно. Никакой полезной информации через неё передать нельзя.
47 585683
>>5681
А можно ли сделать пары пар частиц при запутанности быть похожими и при нарушении запутанности определить, что если бы не разрушили, доминировал бы какой-то спин, а при нарушенной запутанности статистически более или менее одинаковое количество и тех и других спинов?
48 585684
>>5683
Нет и нет. Запутанность только в парах частиц, а параметры полностью случайны.

Никакой информации через запутанность не передается.
49 585686
Поясните как работает краулер на КСЦ? Эта йоба что возила ололон и шаталы. Их две же всего, да?
Вот на нее ракету поставили, она неспешно поехала и заехала на стартовую площадку. Дальше че? Ракета с этой штуки и взлетает, или ее снимают и переставляют на саму площадку? Как?
И для каких ракет эти транспортеры используются, а какие на КСЦ погружают на площадку иначе?
четырехщелевой эксперимент.png23 Кб, 1328x766
50 585693
>>5671
Нет ошибки. Я эту схему ещё в лохматом 2011 описывал. Здесь получаются очень сложные парадоксы, от которых привычная нам реальность идёт по пизде, и появляются всякие приколы с многомерным временем.

>>5684
Пикрелейтед.
Из центрального источника влево и вправо короткими очередями выпускаются запутанные частицы. Каждая очередь это один бит. Слева экран с двумя щелями и справа экран с двумя щелями. Между экранами один световой год, но попадают частицы в них одновременно.
Когда щели свободны - очередь частиц складывается в интерференционную картину. Это "0". Когда хотя бы к одной щели хотя бы на одном экране подносится детектор - с обеих сторон очередь частиц складывается в две полосы. Это "1". Поднося и убирая детектор мы передаём информацию мгновенно.

Суть в том, что нам плевать, в какую щель прошла частица. Нам важно то, разрушена ли волновая функция или нет. Никакого запрета на "дистанционное разрушение" волновой функции нет, потому что волновая функция у запутанных частиц по определению одна единая. Нельзя узнать, что слева частица прошла через щель А, не узнав при этом, что справа частица прошла через щель А'. Интерференционная картина разрушается одновременно на двух экранах.

Здесь вот что важно. Строго говоря, это не является "механистической" передачей информации через причинность пространства-времени. Это нефизический процесс. Тем не менее, такой процесс позволяет совершить видимость полезной передачи информации с точки зрения наблюдателя, что возвращает нас к трудному вопросу, что же такое наблюдатель и что на самом деле декогерирует.
Например, с точки зрения квантового самоубийства сверхсветовая передача данных вполне возможна. Алиса пишет сообщение Бобу, измеряет запутанные частицы и стреляет себе в голову всякий раз, когда результат измерения не совпадает с желаемым битом. Таким образом, в рамках многомировой интерпретации Алиса выживает только там, где Боб получил её сообщение, хотя с точки зрения физики правильный результат у Боба получился совершенно случайно. А описанная выше схема устраняет необходимость стрельбы в голову, как бы выворачивая ящик шрёдингера наизнанку.
четырехщелевой эксперимент.png23 Кб, 1328x766
50 585693
>>5671
Нет ошибки. Я эту схему ещё в лохматом 2011 описывал. Здесь получаются очень сложные парадоксы, от которых привычная нам реальность идёт по пизде, и появляются всякие приколы с многомерным временем.

>>5684
Пикрелейтед.
Из центрального источника влево и вправо короткими очередями выпускаются запутанные частицы. Каждая очередь это один бит. Слева экран с двумя щелями и справа экран с двумя щелями. Между экранами один световой год, но попадают частицы в них одновременно.
Когда щели свободны - очередь частиц складывается в интерференционную картину. Это "0". Когда хотя бы к одной щели хотя бы на одном экране подносится детектор - с обеих сторон очередь частиц складывается в две полосы. Это "1". Поднося и убирая детектор мы передаём информацию мгновенно.

Суть в том, что нам плевать, в какую щель прошла частица. Нам важно то, разрушена ли волновая функция или нет. Никакого запрета на "дистанционное разрушение" волновой функции нет, потому что волновая функция у запутанных частиц по определению одна единая. Нельзя узнать, что слева частица прошла через щель А, не узнав при этом, что справа частица прошла через щель А'. Интерференционная картина разрушается одновременно на двух экранах.

Здесь вот что важно. Строго говоря, это не является "механистической" передачей информации через причинность пространства-времени. Это нефизический процесс. Тем не менее, такой процесс позволяет совершить видимость полезной передачи информации с точки зрения наблюдателя, что возвращает нас к трудному вопросу, что же такое наблюдатель и что на самом деле декогерирует.
Например, с точки зрения квантового самоубийства сверхсветовая передача данных вполне возможна. Алиса пишет сообщение Бобу, измеряет запутанные частицы и стреляет себе в голову всякий раз, когда результат измерения не совпадает с желаемым битом. Таким образом, в рамках многомировой интерпретации Алиса выживает только там, где Боб получил её сообщение, хотя с точки зрения физики правильный результат у Боба получился совершенно случайно. А описанная выше схема устраняет необходимость стрельбы в голову, как бы выворачивая ящик шрёдингера наизнанку.
1A9ktJ8.jpg78 Кб, 439x438
51 585694
Почему негров в космос не запускают? Боятся сравнения с подопытными обезьянами?
Станут ли их запускать после BLM?
52 585695
>>5686
Так прямо с него и запускается, шаттлы и сатурны с MLP стартовали. Да не то блядь, а Mobile Launch Platform. Один транспортер тащит саму ракету, второй - башню обслуживания. Не используется вроде ни для чего, сейчас LC-39 только тяжелые фальконы пускают, а те горизонтально перевозят. SLS скорее всего на краулерах будет ездить уже.
800px-GuionBluford.jpg164 Кб, 800x1000
53 585696
>>5694
А Гайона Стюарта Блуфорда видимо не было, да?
800px-MaeCarolJemison.jpg148 Кб, 800x1000
54 585697
>>5694
>>5696
Как и Мэй Кэрол Джемисон кстати.
55 585698
>>5696
Два уголька среди сотни снежков. Слишком мало.
56 585699
>>5698
Так им никто блядь не запрещает в космос летать, они сами не хотят. И эти двое еще в восьмидесятые летали, когда сегрегацию только-только выключили. Но зачем, если можно сидеть на пособиях и с 12 лет торговать крэком?
57 585701
>>5694
Когда Шаттлы летали - запускали. Летало по 7 человек за раз, по 5-10 полетов в год, могли впихнуть нескольких без особых проблем для миссии. А сейчас, когда пускают по 4 корабля в год, на которых 1, реже 2 американца, как то тяжеловато жертвовать целое кресло ради диверсити и инклюжена.
>>5698
Не 2 а 13.
58 585706
>>5693
О, да это же почти классический delayed-choice quantum eraser.

Одна проблема — твой пикрелейтед в реальности не работает. Интерференционной картины не будет ни в том, ни в другом случае. Вернее, в одном случае будет два перекрывающихся простых горба, а в другой — две комплементарных интерференционных картины, но результат для внешнего наблюдателя будет выглядеть абсолютно идентично. Ни единого бита через такую систему передать нельзя.

>If we instead choose to herald using D1 or D2 without distinguishing between the two, there is no interference pattern. This is because we effectively ignore the polarization state of the trigger photon, leaving the signal photon in a mixed state.



https://www.nature.com/articles/srep04685
Более подробное разжевывание — https://physics.stackexchange.com/questions/179348/double-double-slit-with-entangled-photons
59 585707
>>5695
Башня аполлона не стационарная что ли была?
Как и кто перевозит тяжелые флаконы и как их ставят?
Как тебя отблагодарить за потешную шутейку про млп?
image.png175 Кб, 510x512
60 585708
>>5701

>А сейчас, когда пускают по 4 корабля в год, на которых 1, реже 2 американца, как то тяжеловато жертвовать целое кресло ради диверсити и инклюжена.


Despite making up 13% of the population...
African Americans can't achieve 13% of the astronaut crew.
61 585709
>>5707

>Башня аполлона не стационарная что ли была?


Niet, не стационарная.

>Как и кто перевозит тяжелые флаконы и как их ставят?


Транспортер с подъемником. Тащит горизонтально, подъезжает к стартовому столу, аккуратненько поднимает ракету и мягонько передает в лапки башни обслуживания.

>Как тебя отблагодарить за потешную шутейку про млп?


Я не первый про это шучу, первый раз наверное года три назад здесь же (в ТТВ) увидел.
62 585711
>>5709

>Niet, не стационарная.


А как ее не сносило нахуй при старте сатурна-то? Она ж высокая и узкая...

>Транспортер с подъемником. Тащит горизонтально, подъезжает к стартовому столу, аккуратненько поднимает ракету и мягонько передает в лапки башни обслуживания.


Кто такой, как выглядит?
Вагины-ракетовозы и дизеля на байкодроме я представляю, они примерно одинаковые для всех (обоих двух) ракет что там летают.
А с флоридского огуречника летала половина вариантов ракет мира и я не знаю как они взгромождались на постаменты...

>Я не первый про это шучу, первый раз наверное года три назад здесь же (в ТТВ) увидел.


Точно здесь?
Я тебя постоянно подъебывал ей в треде про СЛС.
bwwnav8l28751.mp490 Кб, mp4,
640x480, 0:05
63 585712
В сайфае любят так три кольца изображать которые вращаются по разным осям.
Есть ли за этим какое-то физический смысл, или это просто идея "округи вращаются по разным осям" и все?
Первый раз увидел в Квейк 2, казалось впечатляюще, а сейчас когда много раз замечал - не вижу смысла.
64 585715
>>5606
анон, ты можешь свои наркоманские измышдизмы подробнее изложить?
65 585716
>>5711

>А как ее не сносило нахуй при старте сатурна-то? Она ж высокая и узкая...


Основная масса - транспортер, а он тяжелый и внизу, как неваляшка получается. К тому же, газы в основном вниз идут, а не в стороны.

>Кто такой, как выглядит?


falcon heavy transporter erector гуглишь и смотришь картинки.

>Точно здесь?


Может и не здесь, давно было.

>Я тебя постоянно подъебывал ей в треде про СЛС


Кого меня то блядь, ты здесь один сидишь. Кроме тебя только бот от макаки.
66 585723
>>5712
Да никакого смысла нет. Это примерно как люди без художественных навыков, пытающиеся нарисовать велосипед по памяти.
http://www.gianlucagimini.it/prototypes/velocipedia.html

Поэтому немалая часть скилла художника заключается в том, чтобы думать как на самом деле устроены вещи, которые он рисует. Чтобы не получилось "ну это хуёмоё рамки гироскопа ебанём давай и сразу вжух такое астро-хуйзнаетчто, чорная дыра тип, или как там оно у ейцегаловых))0".

Да и не припомню что-то нигде, кроме Q2 да может быть Контакта, откуда этот гироскоп ебаный наверно в квейк и передрали.
67 585724
>>5605
Радиация в космосе есть от двух вещей - галактическая (прилетает снаружи гамма-излучение и частицы высоких энергий) и в поясах Земли. В радиационных поясах Земли не собственно радиация, там летают электроны и протоны на больших скоростях, они бахают в корпус аппарата и получается радиация. Вот их отклонить магнитом можно, но нахуй не нужно, пушто в поясе ты все равно не задерживаешься, его облететь можно, и пролетаешь ты быстро. А гамма-излучение - основной источник радиации при перелёте и жизни на Марсе или Луне например - ты не отклонишь магнитом. Частицы высоких энергий можно, но не нужно - они сравнительно редкие, и магнит у тебя получится ебический, жрущий прорву электричества (с генерацией которого в космосе и так проблемы)
Apollo11SaturnVandMobileServiceStructureatpad39-A(482864458[...].jpg3,4 Мб, 3590x3015
68 585727
>>5707
>>5709
>>5711
У Сатурна две башни, одна на пусковой платформе вместе с РКН, вторая сама по себе, и обе ездили на краулерах.
У Флакона так: TEL удерживает ракету, а гидроцилиндры для подъёма встроены в пусковую установку, причём на всех СК конструкции разные. А башен для обслуживания нет.
image.png479 Кб, 675x450
69 585728
>>5716

>К тому же, газы в основном вниз идут, а не в стороны.


У них не выкопана чаша унитаза как у нас на байкодроме же.
Так что внизу идут в стороны.

>falcon heavy transporter erector гуглишь и смотришь картинки.


Гуглится только пикрил - 78 стальных колес которые везут один блок. Я буквально по твоему тексту гуглил. Мне интересно, как ее довозят и водружают наверх.

>Кого меня то блядь, ты здесь один сидишь. Кроме тебя только бот от макаки.


Ох ты дурень, я и есть бот от макаки.
70 585729
>>5723

>Да и не припомню что-то нигде, кроме Q2 да может быть Контакта, откуда этот гироскоп ебаный наверно в квейк и передрали.


Из крупных популярных медиа - считай и не было. А вот в шлаке и пародиях в 90-х предостаточно было. Это как крик Вильгельма в кино или БРРРРРАААААУУУУ в современных трейлерах.
image.png356 Кб, 522x305
71 585730
>>5727
Как ракета с краулера оказывается на этом постаменте?
72 585731
>>5730
Её прямо вместе с постаментом и везут.
73 585732
>>5731
ОХУЕТЬ.
Это ж тяжело дохуя, нет!?
И как они по наклону едут?
И как потом старт происходит, они не боятся сжечь гусеницы у транспортера?
74 585734
>>5732

>И как потом старт происходит, они не боятся сжечь гусеницы у транспортера?


Краулер потом сдаёт назад и выезжает из-под платформы, разумеется. В момент запуска его там нет.
75 585735
>>5732
да, поэтому 2км они едут целый день
там особая система стабилизации, платформа наклоняется при езде в гору чтобы оставаться горизонтальной относительно земли
76 585736
>>5734
Этот момент-то поясни же. Как он сдает из-под платформы? На чем она стоит? Как этот процесс происходит?
Про это видео катастрофически мало.
Как вешают Союзы на подвесы - дохуя, я даже в прямом эфире смотрел, как ставят Протон на стол тоже. А вот как кроулер заезжает, оставляет ракету (как) и уезжает - не видел ни разу.

>>5735
Ни разу не умаляю офигенность этой темы.
Кстати, а почему они рельсы не стали юзать?
77 585737
>>5724

>не собственно радиация


В чем отличие, лол. Радиация - это просто какие-то высокоэнергетические частицы.
78 585738
Какие правила нужно держать в уме, когда пытаешься придумать обитаемую планету? Ничего страшного, что я тут спрашиваю, а не в сайфаче?
79 585739
>>5738
Зону златовласки.
80 585740
>>5736
Вот тут примерно с 11:05 показывают крепления и опоры, к которым мобильную платформу привинчивают.

https://youtu.be/3KOImkgmmk8?t=665

Просто платформа сначала поднята выше уровня опор, потом опускается ниже и встает на опоры, а потом краулер опускается еще ниже и выезжает из-под неё. Тут ничего сверхъестественного нет.
1593172826716.jpg48 Кб, 685x567
81 585741
В первую чашку наливают 50% кофе температурой 90°.

Во вторую чашку наливают 50% кофе температурой 90° и 50% молока температурой 5°.

Через 10 минут в первую чашку тоже доливают 50% молока температурой 5°.

В какой чашке будет выше температура и почему?
82 585742
>>5738
Долговременную стабильность основных параметров. Не должно быть, орбит, звезд, состава атмосферы, которые меняются за миллион лет, иначе хуй там что сложное успеет проэволюционировать.
83 585743
>>5739
>>5742
А по географии что думать?
84 585744
>>5740
У краулера гидравлическая подвеска, спасибо.
Я балбес, сам не додумался.

Охуеть, конечно, уровень работ.

Хотя я даже не знаю, что более ебическое - монтаж платформы над кроулером со стаметровой ракетой над вами, или просто подъем стаметровой ракеты как в СССР.

Кстати - стаметровой или стометровой?
85 585745
>>5741
Недостаточно информации. Попробуйте снова.
86 585747
>>5728

>У них не выкопана чаша унитаза


Насчёт унитазов не знаю, а газоходы есть. Так как большинство РН США стартует с приподнятых мобильных платформ, то они обычно менее глубокие, чем советские, а от срезов сопел до дна расстояния не сильно отличаются.
87 585748
Почему люди ничего не делают?
Арктика почти растаяла, 60% видов вымерло за последние 40 лет, тропические леса исчезают, глобальная температура выросла why?
88 585749
Можно ли космический мусор прямо с Земли хуячить лазером? Какая мощность нужна, чтобы на НОО металл испарять?
89 585750
>>5706

>О, да это же почти классический delayed-choice quantum eraser.


Нет, это гораздо более мощный эксперимент.

В DCQE никакие запутанные частицы непосредственно в экран не летели. Летели производные от них сигнальные фотоны, сами по себе запутанностью не обладающие. Потом они складывались, а интерференционная картина восстанавливалась по производным холостым фотонам.

Здесь же всё иначе. Даун-конвертера нет. Декогерирующего деления на сигнальные и холостые фотоны нет. Есть только удвоенный двухщелевой опыт, работающий с запутанными частицами напрямую.

>Интерференционной картины не будет ни в том, ни в другом случае. Вернее, в одном случае будет два перекрывающихся простых горба, а в другой — две комплементарных интерференционных картины, но результат для внешнего наблюдателя будет выглядеть абсолютно идентично.


Нет, ты не шаришь. Отсутствие интерференционной картины в DCQE обусловлено тем, что в экран попадают уже раздекогерированные сигнальные фотоны. Они не интерферируют на друг с другом, ни сами с собой (ибо щелей перед экраном нет), они просто дают жирное пятно. Просто в этом пятне как бы зашита информация о двух взаимно-аннулирующихся интерференционных картинах, которые мы можем восстановить по регистрации холостых фотонов.

У меня на схеме ничего этого нет. С изначально рождёнными частицами вообще ничего не происходит вплоть до попадания в экран, они просто летят.
90 585751
Где сферы Дайсона?
91 585753
>>5747

>Насчёт унитазов не знаю, а газоходы есть. Так как большинство РН США стартует с приподнятых мобильных платформ, то они обычно менее глубокие, чем советские, а от срезов сопел до дна расстояния не сильно отличаются.


Газоходы есть, но в них водичка наливается, т.к. иначе отдачей ракету распидорасит. Советские унитазы не требуют налива воды, т.к. волна гасится дойдя до чаши унитаза.
Вообще сложно представить себе выкопанные чаши унитазов во флоридском побережье, их бы затопило к хуям.
92 585754
>>5748
Большинство людей - идиоты. Им похуй на мир, они думают только о себе.
У меня нет идей как с этим справляться. У тебя есть?

>>5749
Не особо. Атмосфера справляется сильно лучше, она и лазер тот самый прекрасно будет гасить.

>>5751
Никаких не обнаружено, так что ответ - в фантастике.
93 585755
>>5754
С какого расстояния мы могли бы заменить сферу Дайсона с существующими технологиями?
07.jpg256 Кб, 1118x848
94 585756
>>5732
Не особо тяжело, пустой Сатурн 5 в сборе весит всего лишь 130т, это меньше чем средний карьерный самосвал подымает. (тащемта этот транспортер и есть переделанная платформа от карьерной техники, как у муска переделанные нефтяные платформы для посадки ступеней). У транспортера просто размеры большие, для сравнения вот эта хуйня
https://www.youtube.com/watch?v=1Wp41alE8Mg
может двигать 44 тонны на каждую ось, а собранная модулями в блок она может спокойно целый дом перевезти (в первой половине XX века вообще целые улицы двигали).

Для Н-1 тоже подобную телегу начали пилить, но чтобы ездила не по ровной насыпи, а по степям Казахстана, Зил-135Ш на моторизованных поворотных шасси от Ил-18, пикрилейтед. Вышло бы примерно как на видео, только побольше, не модульная и для бездорожья. Но потом решили делать ракету прямо на месте, и йобу бросили на стадии прототипа.
95 585757
>>5755
Заменить сферу дайсона на что? У нее из альтернатив только кольцо Нивена, наверное.
96 585758
>>5756
Спасибо за развернутый ответ.

>Но потом решили делать ракету прямо на месте, и йобу бросили на стадии прототипа.


А в чем проблема, почему изначально не предусматривалось такое решение? Почему это будто крайняя мера? Это же наоборот наиболее оптимально. Тут же собрал, протестировал и запустил.
97 585760
>>5758
Так собрать ракету это не трубопровод сварить, там высочайшая точность нужна, по части сварки стенок баков переменной толщины, монтажа движков и тд.
98 585761
>>5758
Потому что это использовать имеющийся ракетный завод в Москве vs строить новый, вместе с миграцией большого количества рабочих и спецов. Всё это в пиздец сжатые сроки.
99 585762
>>5760
Ну на Байке на самом деле была отвёрточная сборка из крупных узлов, влезавших в габарит, и сделанных на основных предприятиях, в Самаре в основном. Но да, завод неебовый всё равно получился, туда половина Прогресса переехала. Да и МИК тоже.
100 585764
>>5748
Потому что людям похуй. При этом они не идиоты, просто большинство рассуждают, что чем орать про выдуманные проблемы, луче заняться своей жизнью. Вот и живут. И, кстати, правильно, 90% всей этой алармистской хуеты (в том числе и научных статей по...) или натянуто/выдумано/подтасовано ради карьеры, либо человек идёт на поводу у циничных манипуляторов (в политике/науке) и становится идейным идиотом без критичечкого мышления и уже сам генерирует посты, статьи и разговоры.
Какой пример? "Обилие" статей о пользе глобального потепления (их нет) .
101 585765
>>5760
Это проблема почему?
Для СССР например, в чем проблема переехать станками и спецами поближе к байконуру?

>>5761
А можно строить не целиком новый а завод крупноузловой сборки который уже будет из кучи полезных напиленных деталей делать целиком модули.
У СССР разве была проблема с "миграцией большого количества рабочих и спецов"? Читал про базы на камчатке - там пиздец с нулем инфраструктуры, и довозили всяческими способами все, вплоть до ракет в последствии.
А на байконур не могли куда железка и аэродром нормально ходят?
Что-то не сходится, не понимаю.
102 585766
>>5758

>протестировал


Хех мда. Если б её тестировали перед полётом, она может быть и полетела даже. Но это было бы возможным, если б Королёв не стал городить мемный гиперболоид, а взял бы свою собственную абсолютно нормальную и рабочую вязаночную компоновку от Р-7, разбив первую ступень на блоки меньшего диаметра. Тогда может быть и доставить можно было без ёба-завода в степи, и протестировать блоки по отдельности на маленьком стенде.

Но это я с дивана из 2020 говорю, кто его знает какой там челлендж в 60-х виделся. Может с синхронизацией движков между блоками траблы были бы (они впрочем и так были, борткомпьютер заменяли на более быстрый), может с массой, может с аэродинамикой, может ещё с чем.
103 585768
>>5765

>Это проблема почему?


>Для СССР например, в чем проблема переехать станками и спецами поближе к байконуру?


Дорого. Ракетный завод же не просто отдельно стоит, он зависит еще от кучи предприятий. Да и под спецов надо отдельный город строить со всей инфраструктурой. И все это ради одной Н-1, которая слетала бы пару десятков раз.
104 585769
>>5765
Ну говорить легко, а сроки жмут и мешки ворочать надо. Ну довозили, и собственно так и сделали с Н-1 - завод отвёрточной сборки.
105 585771
>>5766

>взял бы свою собственную абсолютно нормальную и рабочую вязаночную компоновку от Р-7, разбив первую ступень на блоки меньшего диаметра. Тогда может быть и доставить можно было без ёба-завода в степи, и протестировать блоки по отдельности на маленьком стенде.


Подробней, пожалуйста.

>>5768

>>Для СССР например, в чем проблема переехать станками и спецами поближе к байконуру?


>Дорого.


Когда это было проблемой для СССР? Глянь на хранилища боеголовок, на подводные верфи, на массивы антенн в тундре.
106 585772
>>5771

>Когда это было проблемой для СССР? Глянь на хранилища боеголовок, на подводные верфи, на массивы антенн в тундре.


Все тобой перечисленное - это надолго. А проворачивать такую многоходовочку из-за одной ракеты - просто бессмысленно.
image.png193 Кб, 1123x784
107 585773
>>5772
Иронично, не находишь?
108 585774
>>5771

>Подробней, пожалуйста.


А чего тут подробней? Просто увеличенную Р-7 бы запилил, и боковушки тестировал отдельно на меньшем стенде. Н-1 потому и не полетела никуда, потому что её не испытывали перед полётом в сборе. Ну и потому что Мишин оказался долбоёбом. Для такой широченной ступени нужен был ебовый стенд, на него денег уже не хватало, поэтому испытывали по старинке - в полёте. (ну кроме прожига движков по отдельности). У сатурна же наоборот, вложились в испытания по полной, в результате он полетел, а Н-1 нет. Позже в Энергии это всё учли, и она полетела сразу, хоть и не без проблем с оплавленным стартовым комплексом.

>Когда это было проблемой для СССР?


Всегда. Ресурсы только выглядят бесконечными, но такими не являются. Тащемта на Н-1 даже близко столько бабла не дали, сколько Королёв просил, и то не в срок. Всё было именно что перекинуто на срочное клепание МБР, потому что штаты тоже охуели со Sputnik'а, срочно подорвались и наклепали себе несколько сотен Титанов.
109 585776
>>5771

>Когда это было проблемой для СССР? Глянь на хранилища боеголовок, на подводные верфи, на массивы антенн в тундре.


Ты сравниваешь военку с гражданским космосом. Лунный полёт лишь в штатах был общенациональной программой, на которую мобилизовали вообще всю промышленность страны. В совке он никогда таким не был, лишь в 64 году бабла дали побольше чем обычно, но общенациональной программы не случилось.
110 585777
>>5776

>рограммой, на которую мобилизовали вообще всю промышленность страны. В совке он никогда таким не был, лишь в 64 году бабла дали побольше чем обычно, но общенациональной программы не случилось.


Тогда странно как преуспели вообще по всем параметрам кроме высадки человеков на луну.
111 585778
>>5777
Ну потому что муриканцы неторопливо дрочили бибу, планируя где-нибудь лет через 30 полетать, пока ОКБ-1 в темпе клепало свою Р-7, которая к тому же оказалась довольно удачной для РКН (и неудачной для МБР, своей главной задачи). Запуск Жажарина был реализован сравнительно небольшими силами по дальнейшим меркам, просто это было новаторство. А потом они прослоупочили модульные станции, потому что все ресурсы пустили на провальный Шаттл и наяривали на оказавшуюся невыгодной концепцию ручного управления, в результате просрали совкам создание космических роботов и модульных станций, да и вообще буксовали на месте 20-30 лет в пилотируемой космонавтике. Человек, роботы, строительство на орбите, долговременное проживание в космосе - это то, в чем совки преуспели. В АМС штаты смогли значительно лучше, тогда как совки смогли лишь в Венеру, ибо аппараты тупо ломались даже на полугоде перелета до Марса, не говоря уже об аналогах Пионеров. По прикладным тоже сравнения никакого - TDRS, кейхолы и прочий ДЗЗ, ебовый сигинт и т.п., а всё по той же причине, неспособность в длительный срок существования беспилотных аппаратов.
112 585779
>>5778
Я про ключевые моменты космической гонки.
Провал в луне был сдачей в этой гонке.
113 585780
>>5753

>в них водичка наливается, т.к. иначе отдачей ракету распидорасит


Также для защиты от акустической нагрузки и для уменьшения температуры.

>унитазы


У тебя необычная фиксация. Не, правда, раньше такого тут не встречал.

>не требуют налива воды


В пусковой установке семёрок система газовой защиты - эжекция воздуха. В ПУ Энергии водяная завеса. В УКСС, ПУ Зенита-2 и Ангары подача воды прямо в выхлопные струи. В ПУ Протона брезентовые защитные экраны.

>сложно представить себе выкопанные чаши унитазов во флоридском побережье, их бы затопило


Поэтому там стартовые сооружения не заглублённые, а приподнятые.
114 585781
>>5773
Союзы не строят на космодроме, а только соединяют блоки ступеней.
115 585782
>>5780

>Также для защиты от акустической нагрузки и для уменьшения температуры.


Ты ничего нового не написал.
116 585783
>>5781
Тупарь ебаный, ты выбираешь только удобные себе хуйни и споришь с ними, чмо сраное?
Сука, я прав. Смотри еще раз. СМОТРИ.
ДАВАЙ.
Еблан
117 585784
>>5773
Так ступени ракет семейства Р-7 приезжали готовыми, их только соединяли, тому що они в габарит вписывались. А крупные ракеты пришлось бы с нуля делать на месте.
118 585785
>>5779
Ну и я про ключевые моменты. Человек, стыковки, Луна, АМС, станции - вполне были ими. Только шаттлобураны не были частью гонки, пожалуй: шаттл делали не ради рекордов, а в основном ради дешевизны и объемов (а потом он оброс требованиями и получилось то что получилось). А буран делали чтобы воспроизвести возможности, и похуй что там у них получилось.

Хотя прямой гонкой это вообще сложно назвать, каждая сторона намеренно пыталась делать то, что гарантированно не могло в данный момент получиться у другой. Совки например вообще изначально хотели Венеру огибать космонавтами, вместо того чтобы на Луну лететь. Потому что это виделось выгодней в плане обучения методичному изучению солнечной системы, т.к. надо было собирать корабль на орбите, а лунная миссия была в этом плане неинтересная, простой флаговтык. Аналогично со станциями, аналогично с человеком - противоположная сторона либо не хотела, либо была отвлечена и не могла прямо сейчас сделать подобное, либо всё вместе.
119 585786
>>5728

>Гуглится только пикрил - 78 стальных колес которые везут один блок.


О, это ж тот транспортер с видоса >>5756
120 585788
>>5755
Сферу Дайсона обнаружить невозможно. (Разве что если гравитационный ультрателескоп в фокусе солнечной линзы делать и в хайрезе конкретный участок неба рассматривать.)

Из-за сферы Дайсона звезда либо не видна совсем, либо в той или иной степени видна только с полюсов, при этом наблюдаемая яркость звезды не меняется.
121 585789
>>5788
Хер там, возможно. В инфракрасном будет фонить сильно.
122 585790
>>5789
И будет ничем не отличаться от остывающего чёрного карлика или небольшой звезды, скрытой туманностью.
123 585791
>>5790
У остывающего черного карлика светимость очень маленькая будет. А туманность, в которой звезда, сама отражает и рассеивает ее свет, так что это сразу заметно.
124 585796
>>5757
А как насчет полностью разобрать звезду, и жечь её уже самостоятельно в своем реакторе/добыть энергию из её составляющих другими способами?
125 585797
>>5790
Естественных черных карликов еще не существует, если что, их в нашу Вселенную только через пару сотен миллиардов лет завезут.
126 585802
>>5796

>разобрать звезду


Это хотя бы в теории возможно?
127 585804
>>5802
В теории - хули б нет.
128 585807
>>5804
И как? Методы какие?
129 585808
>>5807
Щас бы теория тебе предлагала практические методы.
130 585811
>>5808
Я именно практические методы спрашиваю.
У нас манямирок где все работают в юнификации, похуй на загрязнение, будем хуячить совместно как можем.
Вот вся планета объединена и что она может сделать с солнцем?
131 585812
>>5811

>Я именно практические методы спрашиваю.


Ну, тогда не спрашивай: "В теории возможно?".
image.png26 Кб, 1140x326
132 585813
>>5812
Почему?
Я именно это и спрашиваю, красная часть на этой картинке.
133 585836
Из-за радиации Юпитера, будет ли УКВ-радио работать на поверхности Ио на дистанции 1км, 5км, 10км? Или там будет всё помехами забито? Какая связь там возможна вообще?
134 585837
Если я подпрыгну на Луне, то на какой высоте прыжка вращение Луны подо мной заставит меня приземлиться в стороне от места откуда я подпрыгнул?
135 585846
Почему мы еще не полетели на Марс?
И что там из интересного есть вообще?
136 585849
>>5846

>Почему мы еще не полетели на Марс?


Дораха.

>И что там из интересного есть вообще?


Хуй знает. А лететь, чтобы узнать (роботы могут оче мало) дораха.
137 585850
>>5849

>роботы могут оче мало


Мясной мешок, плес.
138 585851
>>5850
Как там Инсайт, закопался уже хотя бы на метр? Человек бы за пять минут его в землю ногой забил.

За сколько робот смог бы починить крыло лунного ровера с помощью скотча и такой-то матери? За месяц управился бы?
139 585853
>>5851
лол а в реальности космонавты по 2 часа тратят чисто чтобы добраться от шлюза до соседнего отсека
140 585855
>>5851
Как вы заебали тыкать своим инсайтом и буром. Запомните блять, InSight это миссия класса дискавери, т.е. литералли бюджетная (по меркам JPL, естественно) хуйня для ровно одной узкой задачи, из говна, палок и изоленты. Он в 5 раз меньше куриосити должен был стоить на самом деле в 2.5 получилось, как обычно. Он тупо не заточен под универсальность. И ещё представь, сколько тем же людям стоило бы запустить туда человека с этой ногой.

>За сколько робот смог бы починить крыло лунного ровера с помощью скотча и такой-то матери?


Если бы у него был автономный манипулятор - быстро.

Так-то современные UGV на земле вполне себе автономно ездят по пересечёнке, в роверы такая автономность пока ещё не попала, из-за медленного цикла разработки. И я абсолютно легко представляю себе робота, колесящего хотя бы на 20км/ч по поверхности Марса и мапящего всё вокруг. Ясен хуй человека это полностью не заменит, но скажем заSLAMить лидаром место будущей посадки сверху донизу в квадрате 10х10км такая штука может гораздо лучше человека, чтобы не садиться вслепую. Либо летающий дрон.

>>5846
Потому что
- техническая неготовность, надо отрабатывать дохуя чего, прежде чем лететь
- та система, которая может это сделать, слишком размылась за десятилетия, просит слишком дохуя денег, и работает без гарантии результата, поэтому на мегапроекты ей никто не даёт
- та система, которая в теории может сделать быстрее и дешевле, ещё толком нихуя не сделала, и неизвестно, не ожиреет ли пока будет делать

>И что там из интересного есть вообще?


Для быдла - жизнь ищут. А реально - просто удовлетворяют собственное любопытство за чужие деньги. (aka "потому что это охуенно")
141 585856
>>5853
ВКД на МКС это как раз тот случай, где космоняки теряют всякие преимущества перед манипуляторами. Потому что обезьяньи пальцы в перчатках уже не такие ловкие, а манипулятором можно и с Земли и с самой станции управлять с минимальной задержкой, да и набор инструментов получше пальцев можно запихнуть туда. (На МКСовских манипуляторах такого нет, но необходимость в ВКД всё равно сильно сократилась, когда манипуляторы нормально отработали)
142 585858
Сколько нужно времени для создания 1 кг. антивещества с нашими технологиями? Ценой пренебречь.
143 585860
>>5858
Нисколько, сразу килограмм антивещества невозможно создать/удержать в принципе.
144 585861
>>5856

> это другое

145 585862
>>5837
На любой, даже 1мм. Ведь ты не указал насколько ты хочешь переместиться.
Так-то поверхность Луны движется чуть больше чем 4.5м/с.
146 585863
>>5861
Да, полностью автономный робот и телеуправляемый робот - сильно разные вещи
147 585865
>>5797
Но это не точно.
148 585866
>>5860
Я не писал сразу килограмм.
1518959399123.jpg85 Кб, 553x674
149 585867
>>5866
Лол, а как ты тогда собрался считать этот килограмм? Несколько атомов на несколько миллисекунд, затем ещё, ещё и ещё, и таким макаром в сумме насчитать килограмм?
150 585876
>>5865
Ну по крайней мере науке не известны такие процессы, по которым они могли бы образоваться и остыть за время жизни Вселенной. Скорее всего, их и правда нет.
151 585879
>>5836
Бамп вопросу.
152 585880
>>5876

> за время жизни Вселенной.


Есть теории эволюции вселенной, в которых она значительно старше.
153 585881
>>5880
Это не теории, а гипотезы, которые ничем не подтверждены, в отличие от теории Большого Взрыва.
154 585883
>>5853

> а в реальности космонавты по 2 часа тратят чисто чтобы добраться от шлюза до соседнего отсека


Этот пост заставил задуматься о вот чем: бывало же такое, что человек вышел в открытый космос через один шлюз и зашел в корабль/станцию через другой?
Интересует в первую очередь разные входы на космической станции, что вышел тут, зашел в другом месте, но и случаи когда вышел из мира и зашел в шатал тоже интересны.
Какая инфа про это есть?
155 585884
>>5883
Ну как минимум лунные миссии, если не ошибаюсь. Или ты сейчас скажешь, что их не было?
156 585885
>>5855

>Как вы заебали тыкать своим инсайтом и буром.



Просто уж очень смешной обсёр получился, причем даже не сам изначальный фейл, а то, как этот бур целый год пытались поглубже в дырку запихнуть, а он все время обратно выскакивал.
157 585886
>>5855

>И я абсолютно легко представляю себе робота, колесящего хотя бы на 20км/ч по поверхности Марса и мапящего всё вокруг. Ясен хуй человека это полностью не заменит, но скажем заSLAMить лидаром место будущей посадки сверху донизу в квадрате 10х10км такая штука может гораздо лучше человека, чтобы не садиться вслепую. Либо летающий дрон.


Робот будет стоить миллионы долларов просто из-за цены доставки, понимаешь?
Даже джумба на марсе будет стоить миллионы долларов.
А когда у тебя такая цена за килограмм - имеет смысл запихать в этот килограмм максимум пользы.
У тебя там реально может джумба кататься и сметать щебенку пока не ебанется вниз в ущелье валес маринерис.
А ты можешь чуть доплатить и эти же миллионы долларов потратить на то, чтобы запустить туда масс-спектрометр, лазер, обсерваторию, сейсмодатчики и прочий набор инструментов.
И ты, сука, захочешь побольше инструментов запихать, т.к. полет на марс это тебе не полет в париж, это, сука, историческое событие до сих пор.
И ты после этого спрашиваешь.
Ха.

>- техническая неготовность, надо отрабатывать дохуя чего, прежде чем лететь


Инди ракетчики? Да. Роскосмос и НАСА? Опыт есть. Их не сложность останавливает.

>- та система, которая может это сделать, слишком размылась за десятилетия, просит слишком дохуя денег, и работает без гарантии результата, поэтому на мегапроекты ей никто не даёт


Про деньги ты помнишь.
На деле научный выхлоп от спама спиритами был бы охуенный.

>- та система, которая в теории может сделать быстрее и дешевле, ещё толком нихуя не сделала, и неизвестно, не ожиреет ли пока будет делать


Она есть хотя бы на бумаге?

>>И что там из интересного есть вообще?


>Для быдла - жизнь ищут. А реально - просто удовлетворяют собственное любопытство за чужие деньги. (aka "потому что это охуенно")


Жизнь ищут из фундаментальных принципов.
Ареология еще позволяет иметь новый взгляд на геологию. Напрямую "для быдла" непонятно, но затухающая ареосейсмология может помочь более точно предсказывать геологические события.
157 585886
>>5855

>И я абсолютно легко представляю себе робота, колесящего хотя бы на 20км/ч по поверхности Марса и мапящего всё вокруг. Ясен хуй человека это полностью не заменит, но скажем заSLAMить лидаром место будущей посадки сверху донизу в квадрате 10х10км такая штука может гораздо лучше человека, чтобы не садиться вслепую. Либо летающий дрон.


Робот будет стоить миллионы долларов просто из-за цены доставки, понимаешь?
Даже джумба на марсе будет стоить миллионы долларов.
А когда у тебя такая цена за килограмм - имеет смысл запихать в этот килограмм максимум пользы.
У тебя там реально может джумба кататься и сметать щебенку пока не ебанется вниз в ущелье валес маринерис.
А ты можешь чуть доплатить и эти же миллионы долларов потратить на то, чтобы запустить туда масс-спектрометр, лазер, обсерваторию, сейсмодатчики и прочий набор инструментов.
И ты, сука, захочешь побольше инструментов запихать, т.к. полет на марс это тебе не полет в париж, это, сука, историческое событие до сих пор.
И ты после этого спрашиваешь.
Ха.

>- техническая неготовность, надо отрабатывать дохуя чего, прежде чем лететь


Инди ракетчики? Да. Роскосмос и НАСА? Опыт есть. Их не сложность останавливает.

>- та система, которая может это сделать, слишком размылась за десятилетия, просит слишком дохуя денег, и работает без гарантии результата, поэтому на мегапроекты ей никто не даёт


Про деньги ты помнишь.
На деле научный выхлоп от спама спиритами был бы охуенный.

>- та система, которая в теории может сделать быстрее и дешевле, ещё толком нихуя не сделала, и неизвестно, не ожиреет ли пока будет делать


Она есть хотя бы на бумаге?

>>И что там из интересного есть вообще?


>Для быдла - жизнь ищут. А реально - просто удовлетворяют собственное любопытство за чужие деньги. (aka "потому что это охуенно")


Жизнь ищут из фундаментальных принципов.
Ареология еще позволяет иметь новый взгляд на геологию. Напрямую "для быдла" непонятно, но затухающая ареосейсмология может помочь более точно предсказывать геологические события.
158 585887
159 585888
>>5884

>Или ты сейчас скажешь, что их не было?


Блядь, мы же нормально общались, нахер ты меня так оскорбляешь с нуля?
Конечно были.
Но переход астронавтов у них был через каплинг, через шлюзовой отсек, разве нет?

Я в ахуе на самом деле. Я запросто переношу "ты хуесос, мамку ибал" и прочие. Но считать меня противником лунной программы - это ты по больному ударил.
За что, скажи?
160 585889
>>5888
Потому, что здесь утверждать, что пиндосы не летали, и нести всякую хуйню про эфир это скорее норма, чем исключение.
161 585890
>>5889
Да вы охуели в конец.
Репортьте петухов которые такое несут. Каждый репортьте.
Кефиробляди вроде как вполне вне доски.
Пораша да, пиздецкая, надо с ней чего-то делать.
162 585891
>>5890
Кефирщиков банят, да. А поккокмоков, которые визжат про высадку на луне я репорчу, и меня банят, лол.
163 585892
>>5890
В чей конец?
164 585893
>>5891
Надо что-то сделать со спасех тредом (и его антагонистом, само собой).

>>5892
Погоди, "вконец" слитно что ли надо писать?
166 585895
>>5888

>Но переход астронавтов у них был через каплинг, через шлюзовой отсек, разве нет?


Наверно, я скорее всего с советской нелетавшей миссией перепутал.
167 585896
>>5894
Точно. Примеры использования помогли.
Спасибо.
>>5895

>Наверно


Наверное?

>я скорее всего с советской нелетавшей миссией перепутал.


Это с которой?
168 585898
>>5883

>бывало же такое, что человек вышел в открытый космос через один шлюз и зашел в корабль/станцию через другой?


"14 января 1969 года с Байконура стартовал "Союз-4" с Владимиром Шаталовым, а на следующий день - "Союз-5", на борту которого находились командир Борис Волынов, бортинженер Алексей Елисеев и инженер-исследователь Евгений Хрунов. Тогда корабли не имели переходного люка в стыковочном агрегате. Поэтому с борта на борт - через открытый космос."
169 585899
>>5886

>Инди ракетчики? Да. Роскосмос и НАСА? Опыт есть. Их не сложность останавливает.


Опыт чего? Я о том, что в куче аспектов полета к Марсу (автономный полет на много месяцев, в минимальном варианте) даже конь не валялся, принципиально, и никто не чешется. Причем тут ракетчики? Это наименьшая из проблем.
170 585900
>>5889
Че несет, вообще охуеть
171 585901
>>5778

>аппараты тупо ломались даже на полугоде перелета до Марса


Ага, а на Венеру они телепортировались мгновенно. Но так то ты прав, это всё следствия цап-царапинга в электронике, из-за чего собственные разработки начали вылезать из говна только в середине 80-х. И затоптаны ещё глубже в говно в начале 90-х, где и находятся по нынешний день.
172 585914
>>5901
На Венеру лететь в 2 раза быстрей.
173 585923
>>5898
Во, офигенно, то что хотел узнать. А сейчас, в современности такое проводят?
Кстати, при современных ЕВА все с пуповиной вылазят же да?
174 585924
>>5899
Опыт полета к красной планете.
Туда лететь дорого и долго.
Каждая попытка на счету.
175 585935
>>5923

>сейчас такое проводят?


Нет. Нет необходимости.

>при современных ЕВА все с пуповиной вылазят же да?


Только с перецепными фалами.
176 585936
>>5935

>Только с перецепными фалами.


Это совершенно разумно и логично.
У амеров есть SAFER при ЕВА в любом случае.
А у русских?
177 585948
>>5896

>Это с которой?


Пилотируемым полётом на Луну на Л3. Сначала переход из ЛОК в ЛК через открытый космос, притом и через люк в переходном отсеке, соединяющим ЛОК с блоком Д и внутри которого ЛК; потом из ЛК в ЛОК после их стыковки.
>>5936

>А у русских?


Так же, разумеется.
image.png7,2 Мб, 1944x2592
178 585950
>>5948

>Пилотируемым полётом на Луну на Л3. Сначала переход из ЛОК в ЛК через открытый космос, притом и через люк в переходном отсеке, соединяющим ЛОК с блоком Д и внутри которого ЛК; потом из ЛК в ЛОК после их стыковки.


Нихуя себе.

>>А у русских?


>Так же, разумеется.


Почему так же? Скафандры и системы свои же.
Американцы на орланы вешают САФЕР что ли?
179 585953
Меня серьезно беспокоит один вопрос.
А именно, что человечество будет делать, когда уран кончится?
Не секрет, что АЭС - это просто, быстро, надежно, безопасно, удобно и экологично.
Люди будут все больше использовать уран, но он не бесконечен.

Однажды людям потребуется создавать автономные колонии в очень далеких ебенях вроде Плутона, куда энергия от солнца уже не доходит.
Или вообще при межзвездных миссиях.
И вот там-то как раз и нужно питание от радиоизотопов.

Но к тому времени тупые людишки израсходуют весь уран и будут сосать бибу в космосе.
Что тогда делать?

Разве что строить фантасмагорические заводы на Меркурии по искусственному синтезу урана из более легких элементов.
Как эта проблема будет решаться в теории?
180 585955
>>5950

>САФЕР


Аналог для Орлана вроде как есть, но Роскосмос им не особо заинтересован, так что пока не используется.
181 585956
>>5953

>А именно, что человечество будет делать, когда уран кончится?


>Не секрет, что АЭС - это просто, быстро, надежно, безопасно, удобно и экологично.


>Люди будут все больше использовать уран, но он не бесконечен.


1 - Торий.
2 - переработка ОЯТ.
95% оят пригодна для генерации тепла в реакторах.
Ее не перерабатывают т.к. дешевле намайнить свежий уран.

>Однажды людям потребуется создавать автономные колонии в очень далеких ебенях вроде Плутона, куда энергия от солнца уже не доходит.


>Или вообще при межзвездных миссиях.


>И вот там-то как раз и нужно питание от радиоизотопов.


Я очень рад твоему оптимизму. Но я его не разделяю. Современные тенденции показывают, что люди даже на марсе не высадятся в нашу жизнь.

>Но к тому времени тупые людишки израсходуют весь уран и будут сосать бибу в космосе.


>Что тогда делать?


Не израсходуют. Нефть легкодоступную за сто лет израсходуют. Ее цена взлетит. Уран пока что весьма доступен.

>Разве что строить фантасмагорические заводы на Меркурии по искусственному синтезу урана из более легких элементов.


>Как эта проблема будет решаться в теории?


Никак, у тебя очень превратное мнение.
182 585958
>>5955
Космонавты выходят на одной пуповине и фалах не имея реактивной страховки?
183 585962
>>5958
Нет никакой пуповины, всё нужное в самом скафандре.
184 585963
>>5962
Когда перестали пуповину использовать?
image.png1,9 Мб, 900x1200
185 585964
Насколько ярко светятся аккреционные диски чёрных дыр (если это имеет значение — примерно солнечной массы)?
Можно ли их визуально спутать с обычными звёздами?
186 585967
>>5964

>Можно ли их визуально спутать с обычными звёздами?


Это точка которая ярко светится. Можно, конечно.
187 585968
>>5967
Ну там, допустим, зная, что расстояние до ЧД условных 2000 световых лет и масса сопоставимая с солнечной, мы будем видеть её как ещё одну звезду, или она будет неадекватно тусклой (или наоборот яркой) и выбьется из общего ряда?
188 585969
>>5968
Она будет еще одной звездой пока ты не будешь использовать спектроскопию и продвинутые методы исследования.
189 585974
Итак - ищу компанию, для пошива скафандра для выдры
190 585975
>>5974
Докажи серьезность намерений.
191 586001
Отменяли ли запуск шаттла после зажигания ССМЕ?
image.png1,9 Мб, 1886x1686
192 586003
Если бы финансово и политически ничего не мешало, могла ли быть йоба-Энергия?
193 586006
>>6003
То, что было полезно от Энергии стало Зенитом и Атласом-5, а остальное (РД-0120 и всякое с водородом) нужно лишь для сверхтяжа, который после этого был не особо нужен. Единственное что можно было на Энергии-М выводить Мир-2, но вышло как вышло.
195 586012
>>5867
А только на несколько миллисекунд?
196 586023
>>6008
Охуеть.

А вот эти искры, это для поджига водорода да?
Чем искры генерируются, как такая фигня работает? Там же не сидят огромные кремневые крутяшки из зиппо?
197 586028
>>5975
Я назвал выдру - Гагарин
5ff6f84s-960.jpg109 Кб, 537x800
198 586029
>>6003

>Если бы финансово


Ну ты понял. А реально - нет задач при такой стоимости, слишком дорогая одноразовая хрень. Модульность мнимая, как и у Ангары - при изменении нагрузки должно меняться и соотношение масс ступеней, а набирать вязанки с мелкими баками плохо по куб-квадрату. Настоящие модульность и дешевизна могли бы быть у Н-1, вот эту ветку зря забросили. А если бы туда еще и посадку ступеней, эх..
199 586030
>>6023
В Хогвартсе заказывают.
200 586032
>>6003
Экспедиция на Марс.
15754718606540.jpg82 Кб, 1200x1200
201 586042
Является ли толстая жирная атмосфера необходимым условием для жизни на планете?
Если бы не толстая земная атмосфера, не было бы защиты от солнечной радиации, а чем это чревато для поверхности и так ясно.Так же при атмосфере, близкой к нулю жидкости испаряются (и не речь не только о воде)
Значит ли это, что искать жизнь в пределах нашей системы можно только на Земле (логично), каких-нибудь экстремофилов или неуглеродные организмы на Венере, а так же на Титане, большой луне Сатурна? И, возможно, немножнчко Марс,но там вообще тухло в этом плане
202 586047
время оборота планеты, за которое сменяются день и ночь,каким образом влияет на планету?
возмём несколько земель. одна - наша, обычная стандартная. другая вращается вокруг своей оси за какие-нибудь часа 4. третья земля находится в приливном захвате с газовым гигантом, которому приходится луной и, соответственно, один суточный цикл,то есть смена дня и ночь, будет соответствовать месяцу, то есть обороту вокруг своего юпитера, что по времени начинается от пары дней
а на что это влияет? и как?
203 586052
>>6042
Я думаю, что моя выдра - выживет где угодно.
204 586055
>>6047
При прочих равных при долгом дне суровее климат. За день поверхность нагревается, за ночь - остывает. Чем они дольше, тем больше перепады.
205 586060
>>6042
Думаю, что какая-то жижка все же нужна.
Не обязательно вода, но что-то должно быть жидким.
206 586066
>>6060
Метан сгодится?
207 586067
>>6066
Лучше этанол.
208 586074
>>6055

>При прочих равных при долгом дне суровее климат


А вот и наоборот, хоть это и неочевидно. Потоки влажного воздуха стабилизируются, на ночной стороне вечный дождь и оттуда текут реки и моря на дневную, чтобы испариться под солнцем, температура по всей планете выравнивается как в тепловой трубке.
209 586077
>>6074
Это, может, если в приливном захвате со звездой. Если смена суток есть, но длительная, то хуй.
210 586122
А солнце щас жжёт углерод или что-то тяжелее водорода, хоть в каких-то количествах, или нет? И оно щас горячее просто потому, что из-за большей концентрации гелия жечь водород труднее и приходится сильнее греться?
211 586123
>>6122
Там, по-моему, такие "зазоры" между условиями для реакций, что ничего кроме водорода оно жечь не может.

>И оно щас горячее просто потому, что из-за большей концентрации гелия жечь водород труднее и приходится сильнее греться?


Ты имеешь в виду, по сравнению со старыми добрыми временами? Там, вроде, принцип немного другой. Гелия со временем становится больше, поэтому ядро уплотняется и водород сжигается эффективнее и быстрее. Но я не настоящий сварщик, мог и напиздеть.

Алсо, температура внешней оболочки (около 6000 К у Солнца) напрямую не связана с температурой ядра, где, собственно, все реакции и идут. Например, когда Солнце станет красным гигантом, температура оболочки упадет куда-то в район 3000-4000 К, зато в ядре вырастет в разы (а то и на порядки может быть, надо гуглить), чтобы жечь гелий и далее по списку.
212 586124
>>6122
>>6123
Не знаю, насколько это можно считать жжением углерода, но CNO-цикл на Солнце работает, пусть и вклад его в общее энерговыделение что-то около одного процента. Но углерод-азот-кислород там не расходуются, суммарная реакция все равно та же самая — синтез гелия из водорода.

В ОП-посте треда №129 неплохое видео было на эту тему.
213 586127
>>5802
Алё, звёзды сами себя разбирают непрерывно, нужно только этот процесс чуть-чуть ускорить.
214 586128
>>5811

>Я именно практические методы спрашиваю.


Сфера/рой Дайсона с отражающей внутренней поверхностью, например. Звезда перегревается и хлопает, и собирай себе свой водород из свеженькой туманности.
215 586133
>>6127
Наоборот остановить же, чтобы ценное топливо просто так не горело.
216 586137
>>6133
Я говорю про солнечный ветер, который уже выдувает материал из Солнца. При помощи точечного нагрева лазерами этот процесс можно ускорить и магнитными ловушками собирать массу, таким образом на масштабе порядка 10 миллионов лет можно Солнце в красный карлик превратить.
217 586139
>>6137

> Я говорю про солнечный ветер, который уже выдувает материал из Солнца


Это тред тупых вопросов, а не тупых анонов, анон.
218 586140
>>6139
Так что же ты здесь делаешь?
219 586141
Как вообще можно отражать радиоволны от Луны и Венеры? Почему они не поглощаются в грунт Луны и атмосферу Венеры?
220 586142
>>6141
Поглощаются, но далеко не полностью. Тебя ж не удивляет тот факт, что, например, солнечный свет тоже от них отражается, так почему радиоволны должны полностью поглощаться?
221 586143
>>6140

> нет ты!!


А, ну понятно
222 586144
>>6137

>солнечный ветер


Что нам веееетееер, да на это отвееетииит...
223 586145
>>6123
https://ru.wikipedia.org/wiki/Солнце#Солнечное_ядро

>Центральная часть Солнца с радиусом примерно 150—175 тыс. км (то есть 20—25 % от радиуса Солнца), в которой идут термоядерные реакции, называется солнечным ядром[31]. Плотность вещества в ядре составляет примерно 150 000 кг/м³[32] (в 150 раз выше плотности воды и в ~6,6 раз выше плотности самого плотного металла на Земле — осмия), а температура в центре ядра — более 14 млн К.


https://ru.wikipedia.org/wiki/Тройная_гелиевая_реакция

>Тройная гелиевая реакция (тройной альфа-процесс) — цепочка термоядерных реакций, в ходе которой три ядра гелия-4 образуют ядро углерода-12. Она начинается при температуре около 1,5⋅10^8 К и плотности порядка 5⋅10^7 кг/м3.


Как минимум на один порядок.
224 586157
А можно ли послать сигнал с Кьюриосити и принять на Инсайте используя Фобос как рефлектор?
225 586215
А откуда гелий на земле, почему он весь не улетел? Водород понятно - связан, а гей же инертный и легкий
226 586217
>>6215
Альфа-распады радиоактивных элементов, находящихся в природе.
Торий-232 в своем ряду дает 6 ядер гелия на один атом тория.
Уран-238 дает 10 ядер гелия.
Уран-235 в своем ряду дает 9 ядер гелия.
227 586292
>>5265 (OP)
Влияет ли расширение вселенной на уже существующие объекты в плане их размера.
Допустим в максимальной пустоте могучими созданиями создается черная дыра прям вот тютелька в тютельку по массе. Снаружи ставят барьер, чтоб там не долетало ничего.
Вселенная продолжает расширатся и Чд перестанет быть чд?
228 586339
>>6292
Во-первых, нет, гравитационно связанные объекты или тем более сами космические тела в размере не увеличиваются, они как бы «проскальзывают» по растягивающемуся пространству. Заметным влияние растяжения становится только на масштабах сверхскоплений галактик, там уже расстояния настолько огромны, что расширение метрики пространства пересиливает гравитацию.

Во-вторых, ты неправильно представляешь себе, что такое чёрная дыра. Это вообще не тело какого-то определенного размера, чёрная дыра это просто точка, там внутри нихуя нет, кроме центральной сингулярности. Горизонт событий это просто расстояние, на котором пространство вокруг сингулярности настолько искривлено, что даже свет начинает проваливаться к центру ЧД, это не какой-то реальный физический объект.

Никакого влияния расширение Вселенной на чёрную дыру не окажет, т.к. размер ЧД определяется только массой центральной сингулярности и больше ничем.
229 586346
>>6339

>чёрная дыра это просто точка, там внутри нихуя нет, кроме центральной сингулярности


Не надо прям так уверенно заявлять. Никто толком не знает, что там есть. По ОТО там сингулярность, но ОТО не умеет в квантмех, поэтому нельзя быть стопроцентно уверенным в ее предсказаниях.
230 586355
>>6339
Но ведь свет мы видим от галактик которые дальше чем возраст вселенной? То есть летит фотон по плоскости а второй через 1 миллиард лет уже до нас недолетит вроде как, между нами увеличивается?
Тут дело конечно в масштабе, но если вот прям + - 1 кг и черная дыра или не черная дыра и масса не прибавится - ведь в результате расширения пространства ее должно "вывернуть"? Я думаю не мне первому такие мысли приходят, кто-нибудь умный не высказывался?
231 586359
>>6355

>Но ведь свет мы видим от галактик которые дальше чем возраст вселенной?


Они теперь дальше. Пока свет от них летел, пространство расширялось.
232 586360
>>6355

>Тут дело конечно в масштабе, но если вот прям + - 1 кг и черная дыра или не черная дыра и масса не прибавится - ведь в результате расширения пространства ее должно "вывернуть"?



Чёрная дыра может быть любой массы, хоть в 1 килограмм, тут дело не в масштабе. Если материю уже сжало в сингулярность, обратно она уже не разожмётся, это необратимый процесс.
233 586361
>>6360
Хокинг щас перевернулся в гробу.
234 586363
>>6361
Совершенно другой процесс, падающая материя полностью пережёвана и затёрта.
235 586368
>>6361
ЧД дойдет до планковских размеров, в теории, а вот что дальше - никому не ведомо.
236 586425
Итак, готов костюм для выдры. Ищем того, кто сможет запустить. В принципе - на полет 30 минут + 30 минут не так много денег нужно. Скорей всего, выдра вполне спокойно выживет и у меня будет первая в мире космическая выдра
237 586454
>>6425
Выдромучитель, гринписа на тебя нет!
238 586455
Магнитное поле у Земли есть благодаря Луне и её вращению с получающимся магнитным динамо. Можно ли создать магнитное поле и у луны?
А что если бы у нас вместо Луны был бы Меркурий? Было бы у нас более сильное магнитное поле?
А если подогнать Меркурий в Венере у них получится достаточное по силе магнитное поле, чтобы и Венеру и прикрыть и Меркурий активировать?
239 586457
>>6454
Моя выдра будет героем! Запуск через пол годика +-
15923256008340.jpg13 Кб, 200x382
240 586458
>>6455

>Магнитное поле у Земли есть благодаря Луне и её вращению с получающимся магнитным динамо

241 586459
Хочу послушать ваше авторитетное мнение о проекте Lunar Gateway. Насколько перспективным он кажется вам? Взлетит или попил бабла? Насколько он является целесообразным? Критики проекта пишут, что эта станция никому не нужна. И почему еще нет треда по этой теме?
242 586460
>>6459

>Насколько перспективным он кажется вам?


Нужен для того чтобы репетировать долгое пребывание в космосе за пределами защиты магнитосферы земли, которое неизбежно при полётах к Марсу или другим целям в далеком космосе. Так что рано или поздно врата взлетят, может и под другим названием.
Многие уже получили контракты для работы над этими вратами - СпейсХ получила контракт на логистику и разрабатывает корабль грузовик для полётов к Луне, разрабатывают модули, которые в огромной пачке запихнут на ФальконХэви. Разрабатывают канадскую роборуку. Планируют запуск спутника Капстоун, который проверит как оперировать в ангельской ореольной орбите. НАСА и ЕКА уже разработали пакеты экспериментов для присоединения к вратам.
https://blogs.esa.int/exploration/artemis-introducing-ersa-european-experiment-to-monitor-radiation-in-deep-space/

> Критики проекта пишут


Это не критики, а просто хуесосы разных сортов. Один сорт хуесосов это зубрин петух, которому похуй на Луну вообще и на всё что вокруг Луны хотят делать. Другой сорт это скептики, которые не верят что НАСА вообще что-то может, потому даже когда НАСА говорит "мы это сделаем" скептики ноют, что НАСА сказала это несерьёзно и скоро они передумают.

>почему еще нет треда по этой теме?


Потому что станция еще не полетела и кто-то сомневается полетит ли она вообще. А для обсуждения Луны есть лунатред, он в бамплимите, но я перекачу его через пару минут.
243 586461
>>6458
И что не так?
244 586465
>>6461
Все не так, Луна тут вообще никакого отношения не имеет
245 586466
>>6461
Фильм " земное ядро " - там конечно фантастика но про магнитное поле все ок.
246 586467
Можно ли хрюкнуть в космосе?
247 586480
>>6465
Попизди мне тут ещё. Луна с её приливными силами — один из источников разогрева и поддержания в жидком состоянии внешнего ядра Земли, без чего магнитное поле не смогло бы генерироваться.
248 586481
>>6467
Ты так говоришь, словно биосфера Земли со всеми её свиньями и не только находится вне космоса.
249 586483
>>6480
Это ты сам придумал
Аноним 250 586487
>>6480
Ядро земли сварено из сыра
251 586489
>>6487
Как раз из сыра всегда была Луна
252 586491
>>6489
Это устаревшие сведения.
253 586505
>>6467
Если бы кое-кто осилили собственную космическую программу, то вполне смогли бы.
Стикер63 Кб, 250x250
254 586520
>>6339

>это не какой-то реальный физический объект.


Тащемта если достаточно глубоко заглянуть, реальные физические объекты становятся еще более ебанутыми, ЧД в этом плане весьма простая.
image.png87 Кб, 256x256
255 586522
>>6455
Платина.
У луны уже не создашь, как и у винуса, они слишком подостыли. Это когда еще внутри бурлит и крутится - что-то получается, а как застывает - уже все.
По-двачерски подскажу. Ты в мамкином супе можешь сделать воронку крутя его поварешкой, и сможешь поддерживать ее без проблем, пока мамка старым полотенцем не отхлестает. А вот без тебя-луняша, батя сделал свой суп, и в нем ты водоворот не сделаешь, как ни крути, будут только обои отслаиваться.
256 586523
>>6467
Ближайшее к этому совершили китайцы, завезя на орбиту 40 грамм отборнейшей свиной спермы https://www.airspacemag.com/daily-planet/shenzhous-pigs-in-space-8498500/
Видать, хотели особо нажористых свиней так вывести, или новый энергетический напиток для космонавтов.
257 586524
>>6455

>если подогнать Меркурий в Венере у них получится достаточное по силе магнитное поле,


Есть мнение что так и было раньше, но в результате какой-то катастрофы Меркурий перешёл на собственную орбиту. Это объясняет и избыточную плотность Меркурия, и проебанный момент инерции у Венеры, и ее молодую поверхность, а может быть и ее температуру.
258 586525
>>6524

>Есть мнение что так и было раньше


А щас ведь хуй проверишь, вообще невозможно узнать, да?
259 586527
>>6522

>винуса, они слишком подостыли


Мы нихрена не знаем что у неё внутри, только предложения.
Аноним 260 586528
>>6527
Знаем - кусок камня
261 586529
>>6525
На сегодняшний момент никак. Поскольку никаких миссий ни к Меркурию, ни к Венере в обозримое время не будет, то скорее всего и не узнаем.
262 586531
>>6528
/spc на острие науки? Вопрос вулканизма ещё не закрыт, а спейсас уже знает что Вернера- камень.
263 586532
>>6527
А, ну венера горяченькая под шубкой, может и не настолько подостыла.
А вот у Марса глазурь такая, что до нуги и не добраться.
264 586533
>>6529
а бепе?
Аноним 265 586534
>>6531

>ещё не закрыт


Закрыт.
266 586535
>>6534

>полтора мимокрокодивших зонда не увидели прям пукающих вулканов


>вопрос закрыт


Нахуя мы зонды на Марс в поисках воды посылали? Можно же было так же вопрос закрыть.
267 586536
>>6535
Можно, подповерхностные океаны везде есть же
268 586537
>>6536

>подповерхностные океаны везде есть же


Пробурим Луну и купаться будем, да?
Аноним 269 586540
>>6535

>зонды на Марс в поисках воды посылали?


Только в павильонах Голливуда и Мосфильма. Все эти зонды и выделенные на них деньги - просто попилы на уровне государств.
270 586541
>>6540
Сетипетух, ты ли это?
Аноним 271 586542
>>6541

>Сетипетух


Мне все давно понятно - только у стран, где есть какие то киномощности, внезапно есть и "полеты, зонды, космонавты". Нет киномощностей - нет и "полетов", вот и вся программа по космосу.
image.png2 Кб, 163x46
272 586543
>>6542
Ну а мне понятно, куда летишь ты.
Аноним 273 586544
>>6543
Вот! Так и затыкают правду.
274 586546
>>6536
А на Земле есть?
Аноним 275 586547
>>6546
Земля плоская и довольно тонкая, под землей только три слона и черепаха
276 586548
>>6546
Аквиферов полно тащемта, притом много слоев. Про артезианскую воду слышал?
Тащемта один из маняпланов по добыче водички на марсе это и есть прокопать артезианскую и там будет химически ебическая вода в жидком виде.
277 586549
>>6547
Так были ведь киты, не? А они ведь в океане.
image.png461 Кб, 548x640
278 586551
>>6549
[Смеётся из загробного мира]
279 586552
>>6548
Но ведь там, по сути, порода, пропитанная водой. Мокрый песочек, типа.
Аноним 280 586553
>>6549

>в океане.


в варпе
281 586554
>>6552
Есть слои-хуи которые воду не пропускают, глины, скальные породы.
Потому нельзя просто так у нас артезианки копать - воды отличаются от грунтовых, можешь зашкварить и потравить большое количество человеческих жертв, сесть на сгуху и попасть на бабки.
282 586555
>>6551
Так и у него черепаха морская, если что. И если википедия не пиздит.
Аноним 283 586556
>>6554

>просто так у нас артезианки копать


мне заплатили - я выкопал. О том что вода питьевая - не подписывался. Вода только для полива
Аноним 284 586557
>>6551
это нил армстронг ?
285 586559
>>6554
Ну да, поэтому вода из слоев над ними не просачивается вниз. Но ведь это все равно не слой воды (в большинстве случаев, как я понимаю), а слой относительно пористой породы, в трещинах или зазорах между частицами которой сидит вода.
286 586560
>>6557

> армстронг


Луи
287 586561
>>6559
Я думаю, что с определенной глубины идет вечная мерзлота у этих водоносных горизонтов, которая и ограничивает просачивание и рассеивание, а в жидком виде вода на очень большой глубине где тепло и давление позволяют таковой оставаться.
По крайней мере, такое стоит проверить.
Я считаю ареологию очень перспективной и полезной наукой, даже больше, чем поиск жизни на марсе.
288 586562
>>6557
Сэр Теренс Дэвид Джон Пратчетт, единственный человек нормально логично описавший плоскую планету, к модели которой нет претензий даже у самых воинственных апологетов "официальной" науки.
Аноним 289 586563
>>6562

>единственный человек нормально логично описавший плоскую планету


О! Я понял! Это как Паук отлично описал искуственный интелект в своей песне "Компьютер - гитлер"
290 586565
>>6546
Да, недавно нашли на полюсе
291 586578
Анон, скажи пожалуйста, а какие агенства мониторят Гелиосферу на предмет вторжения инопланетных объектов (искусственных и естественных) ?
292 586579
>>6578
Никаких.
293 586581
>>6579
Технологий нет?
294 586583
>>6505
Но ведь те кто может издавать такие звуки и так до последнего времени единственными таксистами были.
295 586584
>>6579
ПРО разве не может их обнаружить на входе в атмосферу?
296 586585
>>6581
Нужды нет.

>>6584
ПРО не следит за объектами у Солнца, смотрят только на землю в ожидании запуска и за объектами на НОО.
297 586588
>>6585
простите, мы видимо друг друга не поняли, Гелиосфера подразумевалось всё то пространство солнечной системы до границ "паузы" воздействия солнечного "ветра" (излучения). А не слежка за солнцем и ближайшей орбиты вокруг солнца.
298 586590
>>6556

>мне заплатили - я выкопал


Рассуждения уровня «мне заплатили — я вылил канистру неизвестной жидкости в городской водозабор». Естественно, подобная отмазка от бутылки не спасает ровно никак.

Без специального разрешения бурить глубже первого водоносного слоя полностью запрещено, неважно для каких целей.
Аноним 299 586594
>>6590
Нал из рук в руки без договора - доказывай что я бурил именно на 70 метров, а не то что я пробурил 3 метра а потом ты с дружками добуривал.
300 586595
>>6561

>Я думаю, что с определенной глубины идет вечная мерзлота у этих водоносных горизонтов


Думаешь ты точно не мозгом.
Пиздец, как ты такой тупой живёшь?
301 586596
>>6584

>ПРО разве не может их обнаружить на входе в атмосферу?


Спутники слежения за запуском ракет противника, смотрят на землю, а не в космос.
Блять, откуда вы такие тупари берётесь нахуй?
Пиздец, мне в такие треды нельзя заходит, я просто охуеваю от уровня тупизны этого скотонаселения.
302 586600
>>6583
Разве что таксистами газа из россии в европу.
303 586601
>>6588
А. Не, государства не следят. Астрономы примерно сами по себе смотрят (за них софт сравнивает фотки) и выискивают новые объекты.
304 586602
>>6594
Так докажут же.
Бур чей? Взят когда и кем? Поехали, объясняйтесь.
305 586603
>>6595
Хорошо ты так себя приложил. Никакой логики, никаких опровержений, просто оскорбление.
306 586604
>>6590
А подобным вообще кто-нибудь занимается?
307 586606
>>6604
Полно, повсюду, даже цены не конские.
308 586607
>>6606
Я имею в виду, кто-нибудь ловит тех, кто бурит незаконно, или всем похуй?
309 586612
А как достигается то что ракета лежит на боку у Союза при старте с Байконура? Блоки же равно заправлены. Стол наклонен?
310 586619
>>6596
А наземные РЛС?
311 586620
>>6619
Загоризонтные же.
Я сам в свое время охуел от технологии, но да, они видят ракетный запуск через половину земного шара.
312 586642
Что произойдет, если направить АМС сквозь кольца Сатурна? Каковы шансы удариться о каменюку? Видно ли сам Сатурн изнутри колец, если смотреть вдоль плоскости экватора?
Аноним 313 586644
>>6642

>сам Сатурн


Всего лишь миф, мы не видим дальше луны.
314 586645
Что у вас тут за шизик новый несмешной?
Аноним 315 586646
>>6645
вот запущу выдру в космос - вы еще обо мне услышите !
12.png38 Кб, 1002x678
316 586647
Кстати, можно колонизировать кольца Сатурна. Че там делать хз (прятаться от вражеских АМС, добывать воду например) но я придумал внутрикольцовый толчкоход, пикрелейтед.
image.png377 Кб, 2090x1282
317 586682
>>6642
Очень малы шансы вредно удариться. Кольца имеют хорошее альбедо, но на деле это песок и льдинки и только очень иногда глыбы.
Тем не менее мы говорим об астрономических скоростях и пролетах, так что шансы на фатальное столкновение велики, ответ на твой изначальный вопрос - АМС убьется либо ударом о крупный камень на орбитальной скорости, либо эродирует песком и тоже убьется.
This kills the crab.jpg.
Поэтому Касиня летал в обход основных колец, пикрелейтед.
Screenshot20200702-191038Bromite.jpg409 Кб, 720x1520
318 586693
Я правильно понимаю, что черные дыры существуют только из-за того, что скорость света - максимальная во вселенной? Ведь если бы это было не так, то обьект, пересекший Г.С., просто ускорился выше скорости света, достиг перицентра и потом снова замедлился до своей скорости. А из-за этого тупого ограничения ему ничего не остается кроме как падать в Ч.Д.

Я правильно понимаю, что радиус Ч.Д. это ее горизонт событий, а на самом деле вся масса сосредоточена в одной точке сингулярности?

Алсо, поясните плез что на этом пике написано. Я то думал, что Г.С. это просто высота орбиты вокруг Ч.Д, скорость на которой равна скорости света. А тут какую-то дичь про гиперпоаерхности втирают. Да еще и Г.С. есть в прошлом и будущем.
319 586696
>>6693

>Я правильно понимаю, что черные дыры существуют только из-за того, что скорость света - максимальная во вселенной?


Да, черная дыра - это область пространства-времени ограниченная поверхностью на которой скорость убегания равна скорости света.

>Ведь если бы это было не так, то обьект, пересекший Г.С., просто ускорился выше скорости света, достиг перицентра и потом снова замедлился до своей скорости.


Ты превратно понимаешь "скорость света". Это не скорость конкретно фотонов. Это фундаментальная особенность пространства-времени, именно с этой скоростью распространяются фундаментальные взаимодействия. Будь она другой - черные дыры бы были другими, например.
"Скорость света" это просто название для предела скорости.

>Я правильно понимаю, что радиус Ч.Д. это ее горизонт событий, а на самом деле вся масса сосредоточена в одной точке сингулярности?


Никто не знает точно, но скорее всего это так.

>Алсо, поясните плез что на этом пике написано. Я то думал, что Г.С. это просто высота орбиты вокруг Ч.Д, скорость на которой равна скорости света. А тут какую-то дичь про гиперпоаерхности втирают. Да еще и Г.С. есть в прошлом и будущем.


Ты конкретней вопросы задавай, что точно тебе разжевать, а не "мненипанятна".
ЧД это именно область, а не какая-то конкретная вещь.
320 586704
>>5693
Кто вообще решил что спин можно поменять? Нет ли такого что просто спин задан с самого начала, и поменяв спин одной частицы с другой ничего не произойдет.
321 586705
>>6704

>Нет ли такого что просто спин задан с самого начала



Эйнштейн, ты? Хули тебе в гробу не лежится?

Локальные скрытые параметры полностью опровергнуты Беллом и его неравенствами, эксперименты чётко показывают, что до измерения никакого конкретного состояния у частицы вообще нет. Вероятностная волновая функция это не просто отражение нашего незнания истинного состояния системы в реальности, это и есть реальность.

Классические механистические объяснения в квантовой физике тупо не работают, буквально другой результат в экспериментах бы получался.
322 586706
>>6705
Как оно на самом деле работает-то? Все идет по прямой просто так?
323 586707
>>5741
Да тут все ясно. Во второй. Если температура окружающей среды выше или равна 47 градусам. То во второй смесь будет еще нагреваться. Если ниже то все равно во второй, потому что более горячая чашка будет терять температуру быстрее. Можно еще площадь чашки посчитать, но думаю этим можно пренебречь.
324 586708
>>6704
Шляпа слетит только.
325 586712
Бамп
326 586739
>>6693

> а на самом деле вся масса сосредоточена в одной точке сингулярности?


Никто не знает, как там на самом деле, ибо существующие теории сингулярность корректно не описывают. Может там шайтаны летают.
327 586749
>>6739
Ну вот, что ты наделал?
Представил себе эту чайхану. Сидят степенные шайтаны-акя и пьют чай. Юные шайтанчики возятся в песке. В кустах арчи шумят майнушки. Ошпаз готовится подавать плов...
328 586761
Может ли черная дыра разорвать нейтронную звезду приливными силами, высвободить нейтрониум, а потом как-нибудь выкинуть на высокую орбиту, где из него образуется маленькая планета, или где оно просто будет плавать?
sage 329 586776
>>6761

>из него образуется маленькая планета


Нейтрониум без ебической гравитации просто взорвется. То есть, вещество можно из НЗ как-то выбросить, но только вот после этого оно перестанет быть нейтронным.
330 586780
>>6776
Если точнее, то не без гравитации, а без давления. Просто в норме гравитация как раз это давление и создает.
331 586783
>>6780
Получается если нейтронку чутка разорвать, ее полностью распидорасит?
332 586785
>>6783
Ну не так, чтобы чутка, но в принципе да. Гугли распад нейтрона, например. Период полураспада в норме всего 9 минут, то есть он охуенно радиоактивный, и распадается на протон и электрон - составляющие обычной материи. Они не будут без давления сидеть плотно прижатыми, пушо электрически заряженные.
333 586787
>>6785
Без гроветации которая бы удерживала дегенератов на месте нейтронка бы стала очень мощно ГОРЕТЬ этим самым распадом нейтронов, ящщитаю.
334 586788
А бывают естественные источники когерентного излучения? Ккой-нибудь магнетар дохуя облучающий горячий юпитер атмосфера которого вынужденно светится, например?
image.png345 Кб, 1280x720
336 586790
>>6789
Ну вот нихуя ж себе, спасибо.
337 586793
>>6749
Это тебе не с этими ЕРЖ пить цикорный эрзац-кофе с свекольным мармеладом и есть гречневые котлеты с ломтиком отрубного серого хлеба с, по случаю прихода дорогого гостя, кусочком рапсового спреда.
205.jpg30 Кб, 542x616
338 586794
>>6793
>>6749
Пожалуйста, продолжайте, у вас какая-то интересная история намечается.
339 586814
Посоветуйте хороший, годный учебник по астрофизике и астрономии, не слишком древний. Можно и на английском языке
340 586857
>>6607

>или всем похуй?


this
341 586858
>>6602
Ты блять прикалываешься штоле?
Люди звонят в ментовку, что убивают, приезжайте спасите помогите. Менты в ответ ржут, что "приедем конечно ваш труп описывать". А ты про то что кто-то будет ловить пацанов с бурилкой.

Соседу два года назад на участке приехали двое пацанчиков просверлили скважину. Метр тыща, 32 метра, 32 тыщи, опускай насос качай.
342 586869
Вы охуели, псы ебаные? Где информация про гей-парад планет? В какое время мск и в каком направлении его искать?
Аноним 343 586892
>>6869
Парад будет что то после 2100-ого
Аноним 344 586894
Итак. Двигатели приехали, костюм готов, запуск выдры будет!
345 586920
Допустим я швыряю тарелочки в атмосфере Земли и на теле с гравитацией Земли но без атмосферы. Где тарелочки будут лететь дальше? Вместо тарелочек можно представить ДВД-диски или пластинки.
346 586945
>>6920
Недостаточно данных. А может у тебя тарелочки гиперзвуковые, а? А?
Но если это обычные метательные тарелочки и они правильно запущены так, чтобы хорошо планировать - в атмосфере пролетят намного дольше, конечно.
347 586959
>>6858
От локации зависит и от рандома.
В каком-нибудь баттхертостане если палок недособирали будут прямо по дворам ездить лицензии на скважины проверять.
348 587127
Челы, я короче долго размышлял и придумал теорию про устройство черных дыр, а так же то, как и где кончается вселенная и почему она не бесконечная, если отталкиваться от теории большого взрыва.
Но так как я шизнутый и не хочу, чтобы какой-то горе ученый-хуй моченый спиздил это. А учитывая 21й век и то, как некоторые любят пиздить всё у других - хочу чтобы авторство было за мной, если реально выстрелит.
Куда податься, чтобы свои шизомысли выплеснуть в эссе и при этом чтобы оригинальное авторство было за мной?
Видос на ютюб? Собственный сайт с одной страницей? Где такие шизобольные публикуются?Кроме двача.
9ygQwFmo-k.jpg204 Кб, 600x738
349 587130
Хало, косманы.
Хочу написать книгу с более-менее твердотой на космическую тему. Если быть точным, очень мне захотелось пройтись по теме системы с несколькими звёздами.
Про то, что бы вы хотели бы почитать в плане сюжетов и историй, это мне конечно же интересно. С удовольствием послушааю ваши манямирки, здравые идеи, вопросы, теории, допущения, предположения и даже видения форм жизни и их условия. На основе это легко смогу сделать парочку историй.
Но сперва надо хотя бы базовую твердоту уточнить, ведь так?
Например, берём для начала вводную: сперва учёные обнаружили далёкую звёздочку в сильнейшие по современным представлениям теле и радиоскопы. Назвали её в честь кого-нибудь, например, Гоголя.
Какк дальше весть номенклатуру?
А Гоголя, Б Гоголя и т.д.?
А если у звезды есть планеты, то они будут, скажем, Б Гоголя 1, Б Гоголя 2, Б Гоголя 3 и т.д.?
А если у планеты Б Гоголя 2 есть луны, как их именовать? Б Гоголя 2а, Б Гоголя 2б и т.д.?
А если у луны Б Гоголя 2а есть своя луна, как тогда еёё обозначать?
А ессли две звезды в ссистеме Гоголя вращаются друг вокруг друга близко, скажем В и Г Гогголя, а вокруг них двоих, по далёёёкой орбите вращается ледяная планета, то как её называть, ВГ Гоголя 1?
А если у В и Г Гоголя есть по парочке пускай и маленьких, но планет, то как тогда считать планету, вращающуюся вокруг общего центра масс В и Г Гоголя, если, наппример, В Гоголя имеет 1 маленькую планетку, а Г Гогголя аж 3? Как тогда считать планету в паре ВГ Гогголя, ту самую далёёкую леедяную?
А может ли быть вообще самая-самая крайня планета, вращаюющаяся вокруг центра масс всех четырёк звёзд системы Гоголя и находящаясятак далеко, что уже почти вышла за пределы гравитационного влияния системы Гоголя?
Оччеь интересно всё это послушать и почитать.

И да, можно предположить, что планета из "Игры Престолов" вращается по переменной траектории между некой обычной звездой и каким-то тёмным карликом, невидимой невооружённым глазом, что может быть че-то от простенькой ЧД, до очёнь чёрной планеты и даже черного или коричневого карлика, что приводит меня к последнему умозрительному вопросу ИТТ - как классифицируются субзвёзды?
9ygQwFmo-k.jpg204 Кб, 600x738
349 587130
Хало, косманы.
Хочу написать книгу с более-менее твердотой на космическую тему. Если быть точным, очень мне захотелось пройтись по теме системы с несколькими звёздами.
Про то, что бы вы хотели бы почитать в плане сюжетов и историй, это мне конечно же интересно. С удовольствием послушааю ваши манямирки, здравые идеи, вопросы, теории, допущения, предположения и даже видения форм жизни и их условия. На основе это легко смогу сделать парочку историй.
Но сперва надо хотя бы базовую твердоту уточнить, ведь так?
Например, берём для начала вводную: сперва учёные обнаружили далёкую звёздочку в сильнейшие по современным представлениям теле и радиоскопы. Назвали её в честь кого-нибудь, например, Гоголя.
Какк дальше весть номенклатуру?
А Гоголя, Б Гоголя и т.д.?
А если у звезды есть планеты, то они будут, скажем, Б Гоголя 1, Б Гоголя 2, Б Гоголя 3 и т.д.?
А если у планеты Б Гоголя 2 есть луны, как их именовать? Б Гоголя 2а, Б Гоголя 2б и т.д.?
А если у луны Б Гоголя 2а есть своя луна, как тогда еёё обозначать?
А ессли две звезды в ссистеме Гоголя вращаются друг вокруг друга близко, скажем В и Г Гогголя, а вокруг них двоих, по далёёёкой орбите вращается ледяная планета, то как её называть, ВГ Гоголя 1?
А если у В и Г Гоголя есть по парочке пускай и маленьких, но планет, то как тогда считать планету, вращающуюся вокруг общего центра масс В и Г Гоголя, если, наппример, В Гоголя имеет 1 маленькую планетку, а Г Гогголя аж 3? Как тогда считать планету в паре ВГ Гогголя, ту самую далёёкую леедяную?
А может ли быть вообще самая-самая крайня планета, вращаюющаяся вокруг центра масс всех четырёк звёзд системы Гоголя и находящаясятак далеко, что уже почти вышла за пределы гравитационного влияния системы Гоголя?
Оччеь интересно всё это послушать и почитать.

И да, можно предположить, что планета из "Игры Престолов" вращается по переменной траектории между некой обычной звездой и каким-то тёмным карликом, невидимой невооружённым глазом, что может быть че-то от простенькой ЧД, до очёнь чёрной планеты и даже черного или коричневого карлика, что приводит меня к последнему умозрительному вопросу ИТТ - как классифицируются субзвёзды?
350 587181
>>7127
Научный журнал
351 587186
>>7127
Сумасшедшие фрики обычно в журналах РАЕН публикуются, там и торсионные поля, и пирамиды из бетона, и разумная вода, твоей статье там самое место будет.
sage 352 587192
>>5858
тыж вкурсе что на антивеществе работать не будет двигатель
Стикер191 Кб, 500x500
353 587208
>>7192
Ну антидвигатель.
354 587237
>>7208
Который будет тормозить, охлаждаться и излучать тьму?
355 587286
Почему бета-излучение может быть как-то опасно? Ну зарядит тебя до какой-то степени, потом из-за статики вообще не будет долетать. Током стукнет о дверную ручку и ладно.
356 587287
>>7286
Почему поток котлет на скорости 10000 км/ч может быть как-то опасен? Ну наешься ты в какой-то степени, потом из-за обжорства вообще воротить будет. Просрешься потом и ладно.
357 587288
>>7286
Ты путаешь обычную электризацию расчёски об кота с ионизирующим излучением, способным распидорасить воздух с образованием атомарного кислорода, да и из других веществ наделать всяких весёлых радикалов.
image.png556 Кб, 540x726
358 587292
О, раз уж про радиацию речь пошла, помогите разобраться с этими радами, греями, рентгенами и зивертами?
Я только беккерель и кюри понимаю, но они не имеют прямое отношение к облучению, беккерель это ебаный герц, а кюри это 10 гигагерц, и ниибет, тупо количество распадов, оно уже дальше считается под нужды.
А вот про болезнетворные живительные эффекты на мясо и железо - как оно все понимается? А то читаю и вижу фигу. То джоуль на килограм, и это джоуль на килограмм, но они разные.
Neutronradiationweightingfactorasafunctionofkineticenergy.gif17 Кб, 482x292
359 587293
>>7292
Грей — джоуль на килограмм. Ничего больше, просто поглощенная доза энергии.

Зиверт — биологический эквивалент джоуля на килограмм, т.е. такое количество радиации, которое надо похавать, чтобы причинить телу тот же ущерб, который бы причинил бы 1 джоуль на килограмм гамма-излучения. Рассчитывается умножением поглощенной энергии в греях на поправочный коэффициент, зависящий от природы радиации. Для альфа-частиц и более тяжелых ядер коэффициент равен 20 (т.е. 1 джоуль альфы в 20 раз опаснее джоуля гаммы), для протонов 2, для электронов 1, для нейтронов целая формула, которая даёт графики как на картинке.

Рад — 1/100 грея. Вышел из активного употребления.
Бэр — 1/100 зиверта. Вышел из активного употребления.

Рентген — более сложная вещь, это такая доза рентгеновского и гамма-излучения, при котором в 1 кг воздуха ионизируется зарядов на ~1/3876 кулона. Устаревшая хуйня без особого физического смысла, применялась только потому, что измерять ионизацию воздуха очень просто, а настоящую поглощенную дозу сложно, поэтому градуированные в рентгенах детекторы радиации были раньше широко распространены. Вышел из активного употребления.

Реально полезных единиц измерения всего две, это беккерель (как сильно фонит источник), и зиверт (как вредно для человека находиться рядом с ним). Остальные единицы это просто архаизмы, которые давно пора забыть нахуй.
image.png230 Кб, 400x350
360 587294
>>7293
Реально 2 только нужны?
Пикрелейтед.

А про свет можно так же? Люмены, люксы, канделы, ниты?

Блжад, как же с джоулями и ньютонами все просто и понятно, а с излучением ебань начинается.
361 587308
>>7293

>Реально полезных единиц измерения всего две, это беккерель (как сильно фонит источник), и зиверт


>измерять ионизацию воздуха очень просто, а настоящую поглощенную дозу сложно, поэтому градуированные в рентгенах детекторы радиации были раньше широко распространены.


Сейчас тупо переградуировали те же счётчики гейгера-мюллера, так что 1 зиверт = 100 рентген.
362 587342
>>7293

>беккерель (как сильно фонит источник)


Беккерель это число распадов в секунду, а распады могут быть ОЧЕ разные. Вплоть до безопасного рутения с его десятком килоэлектронвольт, меньше чем от кинескопа.
TKs6NdhuVQ.jpg47 Кб, 320x240
363 587343
>>7130

>можно предположить, что планета из "Игры Престолов" вращается по переменной траектории между некой обычной звездой и каким-то тёмным карликом, невидимой невооружённым глазом


На эту тему "Задача трёх тел" Лю Цысиня. Коротко - с людьми и даже млекопитающими не взлетит - нельзя предсказать смену сезонов.
364 587409
>>7342

>безопасного рутения


Горжусь Россией!
Стикер63 Кб, 512x285
365 587410
>>7409
Подожди, а как же СВИНЕЦ?
366 587434
Слушал передачу, и там один астроном говорил, что сейчас уже в теории вполне можно создавать зонд, который будет лететь с 1/4 скорости света. То есть он долетит до соседней звездной системы всего за 16 лет.
И я вдруг осознал, что колоссальность космических расстояний сильно преувеличена. Если наша цивилизация существует всего 4 тысячи лет, и мы уже можем создавать такой аппарат, который от одного конца галактики может долететь до другого всего за 400 000 лет, то для цивилизации, которая существует скажем 100 000 лет вообще не будет никакой проблемы открывать планеты скажем с водой, посылать туда зонды с нанороботами, генетическими кодами, которые будут разворачивать на планетах генетические лаборатории, базы, колонии, и всё это будет происходить в абсолютно божеские сроки, и весь процесс будет автоматизированно воспроизводиться и зонды будут с тех планет отправляться дальше и дальше.
И если уже сейчас 1 гипотетический зонд может пролететь галактику всего за 400 000 лет, то такая цивилизация практически наверняка заселит вообще всю галактику за несколько миллионов лет. Ну пусть 10 миллионов лет, но это даже в пределах существования одного вида вполне приемлемый отрезок времени.
367 587435
>>7434
Примерно так почувствовал?
368 587436
>>7434
а нахуй это нужно вообще?
369 587437
>>7434

>уже в теории


Именно что теории, а не практики.

>наверняка заселит вообще всю галактику за несколько миллионов лет.


Уже придумали, известно как зонд фон Неймана.
370 587438
>>7436
Ну это просто свойство жизни. Я понял вдруг, что в этом нет никакой фантастики и полное заселение галактики одним видом, при чем не какими-то абстрактными ИИ, а именно биологическим видом вполне реально в весьма короткие сроки.
371 587439
>>7437

>Именно что теории, а не практики.


А какая разница? Там ограничений никаких фундаментальных нет, просто я так понял расчеты какие-то сложные, но это именно уже не фантастика, а реальность. Зонд маленький и к нему будет прилагаться большой парус. Это просто будет реализовано через некоторое время, но факт в том, что в этом нет никаких проблем с точки зрения теории. А значит с развитием технологий и практических не будет проблем.
372 587442
>>7439

>ограничений никаких фундаментальных нет


каждый раз когда я встречаю такое выражение далее идет отборный бред
373 587443
>>7442
Это же не я говорю, а астроном, забыл его фамилию, он часто выступает. Ему наверно виднее.
374 587444
>>7439

>я так понял расчеты какие-то сложные


>Зонд маленький и к нему будет прилагаться большой парус.


Ты про "лазерную тягу"? Если так, то одна из проблем будет в том что лазерный луч будет расходиться слишком быстро. Это основы оптики, там те же принципы что формируют диаграмму направленности у параболических антенн или ограничивают разрешающую способность телескопов. И множество других сложностей, увеличивающих затраты на подобное до бесконечности.

>А какая разница?


Разница в том, что когда это станет возможно практически, наверняка все уже изменится, возможно включая теорию.
375 587448
>>7444

>наверняка все уже изменится


Наверняка в лучшую сторону. Енивей явно существует такой простой способ экспансии, и явно высокоразвитая цивилизация, прошаренная в робототехнике и генной инженирии будет способна его реализовать, и заселить без всяких варп-двигателей всю галактику в разумные сроки.
376 587449
>>7448

>явно высокоразвитая цивилизация, прошаренная в робототехнике и генной инженирии будет способна его реализовать, и заселить без всяких варп-двигателей всю галактику в разумные сроки


Но этого не произошло.
Все потому что хуманы телескопом своим воюют не в ту сторону.
Потому что надо не вверх, а вдоль внутрь.
377 587452
>>7343
Читал. Но там, объективно говоря, задача четырёх тел, ведь мы рассматриваем планету цивилизации трисолярцев как тоже тело, а это +1 к тройной звезде
378 587453
>>7452
Тем более, надо учитывать, что ЗТТ рассматривается, насколько я помню, для тел одинаковой массы и разщмера на примерно равно расстоянии, а в мире гипотетического мира "Игры престолов" естьтолько 11 звезда-светила, один крупный гравитационные незримый объект и сама планета с ИП, крутящаяся вокруг звезды-светила
1200px-RocheLobesDetailed.svg.png297 Кб, 1200x1253
379 587484
>>7453

>ЗТТ рассматривается, насколько я помню, для тел одинаковой массы и разщмера на примерно равно расстоянии


Нет, тебе траектории и в системе Земля-Луна прекрасно расколбасит, если будешь лететь слишком медленно в окрестностях точек Лагранжа.
380 587486
>>7208
при анигиляции позитрон-електронной пары, происходит рождение гамма квантов, у человечества (и во вселенной) нет зеркал способных отражать гаммма кванты
381 587487
>>7486

>електронной


Хiхiл, спок.
sage 382 587489
>>7487
я хихил йобаный - мне можно
e6537609d8a5bea10611c345b351c7fd.gif758 Кб, 500x500
383 587514
>>7486

>нет зеркал способных отражать гаммма кванты


А и не надо их отражать, надо вывернуть надутый пузырь Альбукерке так, чтобы всё излучение из него ебашило в одну сторону.
384 587536
>>7486

>при анигиляции позитрон-електронной пары, происходит рождение гамма квантов, у человечества (и во вселенной) нет зеркал способных отражать гаммма кванты


Во-первых не факт.

Во-вторых гамма-кванты не обязательно отражать, их можно с одной стороны поглощать и конвертировать во что-то полезное. Например, установку по добыче электричества из гамма-квантов с близким к 100% кпд я в прошлых тредах описывал. Это электричество идёт на лазер, тем самым получается функциональный аналог зеркала.
385 587562
>>7486
Ну поглощай их хули, тоже импульс.
386 587576
>>7449

>Но этого не произошло.


Ну возможного этого не произошло из-за ничтожной вероятности именно зарождения жизни в галактике, и мы будем первыми, кто её заселит, раз это так просто и быстро можно реализовать.
387 587579
>>7514
Ты его надул, чтобы выворачивать?
388 587585
Не могу понять/представить четырехмерное пространство, пиздос... Памахите! У меня видимо в голове, что-то не работает. На вас вся надежда!
389 587587
>>7585
А волшебного средства понять и нет.
Тебе надо много и упорно работать над этим.
390 587591
>>7587
Каковы ваши предложения? пойти нахуй
391 587599
>>7591
Я же сказал - упорно работать над этим.
Ну если конкретно, то например, разработка собственного 4д рендера отлично поможет.
VID20200610160913554.mp41,9 Мб, mp4,
384x480, 0:15
392 587608
>>7599
На этом мои стремления в познании паходу всё.
393 587610
>>7434
Ещё 26 тысяч лет осталось подождать.
394 587650
>>7434
Цивилизация подразумевает связность, иначе это не цивилизация, а кучка изолированных островов с папуасами, где каждый папуас заперт в пределах острова, и всякое сообщение между островами отсутствует.

Мы в любом случае обречены дрочить низкоуровневую природу пространства до тех пор, пока его не порвём и не сделаем сверхсвет. Неделимый атом порвали - значит и пространство порвём. Надо всего лишь хорошо (и в правильное место) стукнуть.
395 587651
>>7650
Да, иначе это кучка разных цивилизаций, которые со временем станут разными видами.
396 587652
>>7650
Я тебе по голове стукну. Законы вселенной нельзя нарушать, иначе накажут.
397 587654
>>7651

>станут разными видами


И чо? Разные виды вполне могут работать вместе как одна цивилизация. Мы же как-то уживаемся с неграми и хохлами)))))
image.png158 Кб, 1280x720
398 587655
>>7652
Как что-то плохое. Вселенная все равно равно или поздно кончится, так пусть с лулзами.
image.png1 Мб, 1438x1080
399 587656
>>7654
Touché.
400 587658
>>7655
Ты охуел кончать вселенную - это как огромный корабль на котором 1000 людей в своих каютах сидят, не могут друг с другом поговорить из-за переборок(у нас расстояния), но один пидор решил что он один такой и решил пробить дно корабля, утянув на дно не только себя, но и всех остальных. Тебе бы понравилось если бы сейчас всленная начала исчезать и ты бы даже не понял почему это происходит, а произошло это потому что в 1000000 миллиардах световых лет какая-то цивилизация морских ежей порвала ткань пространства и времени ради лулзов или эксперимента, и таким образом убила вселенную для всех?
401 587660
>>7658
Да ладно, не кипятись. В другой вселенной попробуем снова.
402 587661
>>7650
Меня просто возбудило осознание, что это реализуемо теоретически в самые короткие сроки без всякого сверхсвета, и при этом не реализовано фактически судя по всему. Хотя галактика существует миллиарды лет.
403 587662
>>7660
да я не кипячусь, тот факт что вселенная не умирает и не светится всякими неестественными цветами как раз доказывает, что она почти неразрушима и что разум редок. Планет с тваринами может быть дохера, а там где животные учатся технологиям нихуя. Прям миллионы планет со своими птицами, растениями, рептилиями, динозаврами, которые живут, жуют, срут, но ничего более и так пока их звёзды не дохнут.
end times.webm3,4 Мб, webm,
256x256, 1:59
404 587663
>>7658

>Тебе бы понравилось если бы сейчас всленная начала исчезать


Да, я не против. Охуенно было бы если бы еще это играло. Я уже смирился с конечностью бытия.
405 587665
Был спор по поводу того, входят ли другие туманности в нашу галактику. И одним из аргументов спора заключался в том, что в какой-то момент в туманности наблюдался взрыв сверхновой, который был неправдоподобно ярким, то есть если бы туманность была целой галактикой, то он бы не смог бы перебить её свет.

Я никак не могу найти инфы по этому поводу, как этот взрыв, который закрыл светимость целой галактики объясняется сейчас? Как это возможно?
001.jpg399 Кб, 809x1106
406 587689
>>7652>>7658
Пикрелейтед. Вселенная кончилась ещё тогда, когда первый безумный Тумба принёс в пещеру факел, которым можно сжечь весь лес!
407 587699
>>7665
Сверхновые легко перекрывают яркость всей галактики.
image.png665 Кб, 652x652
408 587708
409 587709
А может ли гипотетически работать звезда на реакциях распада, состоящая из тяжелых элементов?
Планету из кротов собирали, что будет, если звезду из природного урана собрать?
410 587718
>>7689
Вот ето рофел.
411 587775
>>7709
Из природного урана она нахуй взорвётся, ещё не родившись. Даже из 238.

Нужен какой-то едва фонящий элемент, чья критическая масса соизмерима с массой звезды. И для начала надо этот элемент в товарных количествах и в высокой концентрации где-то взять.

А так-то тлеющие ядерные реакции тяжёлых элементов в недрах планет идут, в какой-то степени это и есть твоя звезда.
412 587810
>>7775

>нахуй взорвётся


Так звёзды и так взрываются же. А гравитация их удерживает.
У наших газовых звезд есть предел массы выше которого гравитация уже не удерживает рассеяние разогретой массы из-за энергетики происходящего.
У звезды из какого-нибудь урана будет уже скорее предел в виде радиуса шварцшильда, если докидывать уран, то с какой-то массы она становится черной дырой. Уран сильно плотнее, а еще менее энергетичный, чтобы урановая плазма слишком далеко разлеталась.
В итоге если посидеть и посчитать можно прикинуть параметры этой искусственной звезды, ее примерный график и жизненный цикл.
Звучит как прикольный мюсленный экскремент. Надо Рэнделлу Манро написать, может поугарает.
unlockstephenhawking.png54 Кб, 262x338
413 587827
Хокинг был полностью парализован. Сохранялась ли у него чувствительность конечностей и тела? Или его можно было подвесить как пиньяту и пиздить палками, а он бы ничего не почувствовал?
414 587830
>>7827
АЛС задействует только моторную часть коры мозга.
415 587855
>>7810
Он же умер.
416 587856
>>7855
https://en.wikipedia.org/wiki/Randall_Munroe
Википедия не в курсе.
417 587860
>>7856
Ну просто what-if.xkcd.com не обновляется.
hellohtmlm14d5b2cb.jpg127 Кб, 880x528
418 587861
Итак, предварительный запуск выдры на 8 октября
419 587863
>>7860
Да, он там охуел походу совсем.
Надо его ИРЛ найти и отмудохать со словами "где апдейты вотифа, где, сука?", и сказать, что если не будет хотя бы раз в месяц апдейты постить, то сломаем ноги.
Кто возьмется за такую задачу?
420 587889
Возможно ли по аналогии с воздушными шарами в атмосфере Венеры "Вега-1 и 2" запустить в атмосферу Юпитера зонд на воздушном шаре? Возможен ли полет воздушного шара в разреженной атмосфере Марса?
421 587890
>>7889
На юпитере ветра ебические жи.
А на марсе хуй, можешь попробовать жидким вакуумом заправить.

Сверхлегкий вертолет зато будет летать, да, физика позволяет.
422 587891
>>7889
на юпитере нельзя так как там атмосфера водород, а легче него ничего нет
на марсе можно в теории, но очень невыгодно
423 587893
>>7889
В теории может летать монгольфьер с нагретым водородом. На практике же он должен быть очень большим (т.к. g большое), да и просто его страшным ураганом распидорит к хуям.

Также может летать аппарат тяжелее воздуха, а энергию брать да хоть из того же градиента ветра, были такие концепты - два планера, связанные тросом, в слоях с разной скоростью. Или тупо планер, ловящий термалки, хуялки или что там есть восходящее, 100% есть. На практике опять же планерам и самолётам тупо пизда, уж слишком буйные процессы прлисходят из-за более сильной гравитации и необъятности дедули Юпа.
424 587899
>>7893

>g большое


Ващет, это строго похуй. Во сколько раз вырос вес нагрузки, во столько же выросла и подъемная сила. Большим он должен быть потому, что вокруг тоже водород, ненамного плотнее того, что внутри.
425 587900
>>7827
Учитывая что он был два раза женат и у него было трое детей, какая-то чувствительность где-то по-люблму оставалась
426 587901
>>7900
Последнего ребенка, Тимоти, он заделал в 79 году, он только тогда стал вынужден пользоваться креслом.
А для брака рабочий ШТАНГЕНЦИРКУЛЬ не является необходимым. Хотя технически он да, мог работать.
427 587905
Мальчик, музыка ...
Во-первых, Алькубье.
Во-вторых, он не падает, но вы строите ковчег изнутри (иначе вы не родитесь).
В-третьих, НАСА управляло тысячами самолетов
Это точка доступа для больших проектов, как в прошлом.
Рак Антарктиды.
thinking emoji corrupt.gif455 Кб, 256x256
428 587906
>>7905

>вы строите ковчег изнутри (иначе вы не родитесь)


Как проницательно.
15703555978660.jpg19 Кб, 604x453
429 587944
>>7650

Пока звезды погаснут времени куча

Может даже удастся сбежать за пределы Вселенной

Ей то пиздец все равно
430 587960
>>7944
Надо найти Очко Вселенной, которое старше ее самой и заныкаться в нём.
431 587965
Почему у нас большая полуось? Есть малая полуось?
Почему у них полубольшая ось? Есть полумалая ось?
432 587978
>>7965
Есть. Есть.
433 587979
>>7978
Бред, это просто среднее значение орбиты, оно одинаковое в любом случае, это перигей поделенный на апогей, неважно что на что делишь - значение половинное одинаково всегда
435 588022
>>7130

>А если у луны Б Гоголя 2а есть своя луна, как тогда еёё обозначать?


не бывает лун с лунами

>А ессли две звезды в ссистеме Гоголя вращаются друг вокруг друга близко, скажем В и Г Гогголя, а вокруг них двоих, по далёёёкой орбите вращается ледяная планета, то как её называть, ВГ Гоголя 1?


не уверен, что на таких расстояних, даже при относительной близости звёзд, возможна общая планета

>А может ли быть вообще самая-самая крайня планета, вращаюющаяся вокруг центра масс всех четырёк звёзд системы Гоголя и находящаясятак далеко, что уже почти вышла за пределы гравитационного влияния системы Гоголя?


это ещё более невероятно

>Про то, что бы вы хотели бы почитать в плане сюжетов и историй, это мне конечно же интересно


поиграв ввов в своё время задумался, как на планете может быть несколько разумных доминирующих видов. напиши и обоснуй, какие у тебя в истрии есть разные виды и обоснуй, почему они там и почему они разные
436 588036
На чём работает электроновский Kickstage? На вонючке или керосине?
437 588037
>>8036
Говорят про "зеленый" маняпропеллент.
Если не дебилы, будут гидразин юзать, он не зря прошел проверку временем блин.
438 588053
>>8037
Может быть, кучерявый пиздит, и под названием экологически чистой хуеты скрывается обычная вонючка?
439 588054
>>8053
А хули такого особо грязного в ММГ? Качественные и практичные виды топлива.
440 588057
>>8054
Загугли токсичность гидразина и его производных
441 588073
>>8037
>>8053
>>8054
Дураки штоле?

>>8036
HAN+ADN, любая смесь с закисью азота, перекись водорода, что угодно может скрываться под этим, пока не раскроют не узнаем. Точно не ALICE, лол.

HAN+ADN звучит как наиболее подходящий вариант, т.к. они по всем параметрам лучше гидразина, кроме масштабирования - не подходят для больших установок. Электроновский кикстейдж совсем маленький, по сути это компактный РБ с компоновкой Фрегата и намного меньшими масштабами, движок на 120 ньютон, там явно что-то с человеческим импульсом, а не перекись. И на криогенику тоже непохоже.
442 588074
>>8053
Тогда бы понадобились допуски, оборудование, сертификации, и вообще с гидразином ебли не оберёшься, что сильный перебор для такой мелкоракеты. Ззеленые топлива юзают из-за простоты, в поле заправил, с грузовика запустил.
443 588075
>>8022

>не бывает лун с лунами


Ничто не запрещает им существовать же.
444 588078
>>8073
Хотя везде пишут что там некая однокомпонентная херня. Значит HAN+ADN и криогеника точно отпадают. Либо смесь какой-нибудь хуйни с закисью, либо перекись и катализатор, либо каталитическое разложение какой-нибудь производной ADN как вариант, например
445 588102
>>8057

>токсичность гидразина


Так нехуй его пить, блядь. Его в ракете сжигать надо.
maxresdefault.jpg16 Кб, 1280x720
446 588126
>>8102
А если хлопок?
447 588129
>>8126
Если площадку не распидорасило то похую.
1594472983371.jpg572 Кб, 1671x1086
448 588181
Как называются эти маленькие крылышки в передней части самолёта?
450 588183
>>8182
Спасибо, но было вроде какое-то русское название
451 588184
>>8183
Ты ждешь слова типа "антикрыло" или чего?
452 588208
>>8181
Переднее горизонтальное оперение (ПГО)
0000291594311821397982big.jpg93 Кб, 870x486
453 588221
Местные долбоебы вообще следят за событиями в космосе или нет? Посбивались по шконкам и сидите надрачиваете, пока по небу кометы летают, а планеты проводят парады
454 588229
>>8075
назови известные луны сосвоими лунами
455 588232
>>8229
Луна
456 588233
>>8229
я слышал на сатурне у кого-то есть
457 588236
>>8221
Комета хуйня какая-то, едва видимая невооруженным глазом, а парада планет и вовсе не было — там угол между планетами в 100+ градусов.
458 588241
>>8232
ага, минмус наверное
>>8233

>я слышал на сатурне у кого-то есть


назови
459 588255
>>8241
Вокруг Реи стабильные орбиты могут быть, и там даже подозревали наличие колец, но на фотках с Кассини нихуя не нашли.
460 588273
>>8229
Перечитай ещё раз на что отвечаешь.
461 588275
>>8102
И гидразин и АТ, внезапно, надо синтезировать, перевезти и заправить. Для этого нужны пиздец какие предосторожности, это без пяти минут химическое оружие. Допуски, обучение, спец. оборудование и отдельное здание для заправки с полной фильтрацией, сертификация, трудовые гарантии, специальная подготовка пожарных и т.п. Приближаться к севшему аппарату с вонючкой могут только спецлюди, а любой потенциальный контакт с ним должен быть задокументирован. Если падает истребитель с гидразином в ВСУ, тот же F-16 например, там целая спецоперация по чистке унитаза вилкой идёт, хотя там его хуй да нихуя. С вонючкой столько ебли, что от неё с радостью избавятся, если будет подходящая альтернатива.

>Его в ракете сжигать надо.


Ракетные двигатели не сжигают 100% топлива, они работают с избытком по горючему, выбрасывая определённый процент наружу. Определённое количество вонючки остаётся ещё и в трубах в отработавшем аппарате, актуально для нижних ступеней.
462 588288
Вопросы такие. Если на какую-нибудь любительскую ракету петарду поставить электрический ракетный двигатель сопоставимого с этой ракетой размера, на какую высоту сможет подняться ракета?

Можно ли электрический ракетный двигатель поставить на планер? Летать будет?
463 588289
>>8288

> Если на какую-нибудь любительскую ракету петарду поставить электрический ракетный двигатель сопоставимого с этой ракетой размера, на какую высоту сможет подняться ракета?


Не оторвется от земли.

>Можно ли электрический ракетный двигатель поставить на планер? Летать будет?


Можно. Будет планировать как и планировал. ЭРД на высоте где-то 250 км способен компенсировать драг атмосферы. Ниже - не способен, его миллиньютоны тяги никакой погоды не играют.
464 588290
>>8289
То есть в тропосфере и стратосфере электрический ракетный двигатель бесполезен. Спасибо за ответ.
465 588291
>>8290
Он не способен дать заметную тягу при внешнем давлении.
Поэтому их испытывают исключительно в вакуумных камерах.
466 588292
>>8289

>миллиньютоны тяги


Теоретически возможны электронагревные двигатели с большой тягой, например электродуговые. Практически для них батареек нет, поэтому исследования почти не ведутся.
467 588293
>>8292
Когда ты в атмосфере тебе для движения достаточно задействовать эту самую атмосферу. Например, пропеллером.
468 588294
>>8288

>Если на какую-нибудь любительскую ракету петарду поставить электрический ракетный двигатель сопоставимого с этой ракетой размера, на какую высоту сможет подняться ракета?


У ЭРД всё зависит не от размера двигателя, а от количества электроэнергии, которое ты готов приложить. Для понимания масштабов, мгновенная мощность, выдаваемая всего лишь одним ЖРД Merlin на полной тяге - порядка гигаватта, это мощность большой электростанции. Такой источник энергии на ракету не запихать, а если и запихаешь, он быстро разрядится. Плотность хранения электроэнергии намного меньше плотности химических топлив. (не считая ядерных источников, естественно)

Кроме того, при одной и той же мощности ты должен выбирать - удельный импульс или тяга. Если хочешь и то и другое сразу, нужно увеличивать мощность.

>>8290
>>8291
Зависит от типа ЭРД. Ионные двигатели способны работать только в глубоком вакууме. Магнитодинамические ЭРД в принципе могут и при давлении работать, хоть и с низкой эффективностью, т.к. это банально плазменная горелка с микроволновой печью, производящей плазму, и магнитом, её разгоняющим. А тем более электродуговые. МГД-двигатели (не плазменные, разумеется) даже в лодки ставили, ради хохмы эксперимента.

>>8289
ЭРД это далеко не только двигатели Холла, да и они способны выдать ньютоны и больше - знай только питай. На практике даже ионники бывают на десятки киловатт, при этом . Опять же, плазменные движки могут иметь на порядки большую тягу, сравнимую с химическими, и динамически разменивать тягу на импульс по желанию (см например VASIMR)

>>8292
Электродуговые двигатели используют в гиперзвуковых аэродинамических трубах, например.

>>8293
Это уже будет не ракетный двигатель. ты только что Rotary Rocket
469 588295
>>8294

>На практике даже ионники бывают на десятки киловатт, при этом выдавая единицы ньютон


фикс
470 588296
>>8294
Очень хороший пост, спасибо.
i.jpg23 Кб, 344x320
471 588297
>>8294

>Плотность хранения электроэнергии намного меньше плотности химических топлив.


Это не догма. Так-то аккумуляторы тоже химические источники, и уже подходят к этим пределам. Другое, что если использовать эту энергию для нагрева рабочего тела, получится пикрелейт.
472 588298
>>8297

>Так-то аккумуляторы тоже химические источники, и уже подходят к этим пределам


Нет. Одноразовые химические источники энергии по прежнему на порядок менее энергоемкие чем прямое сжигание солярки.
473 588299
>>8297
Имеется в виду сколько ты получишь электроэнергии на 1кг аккумуляторов, и сколько тепловой от сжигания 1кг топливной пары. Разница там в районе порядка, если не больше.
474 588300
>>8298

>порядок менее энергоемкие чем прямое сжигание солярки.


Окислитель не забыл?
475 588301
>>8300
Нет, по прежнему порядок.
476 588302
>>8301
Берём водород, кислород, соединяем в топливном элементе, почему на порядок меньше энергии получится чем сжигать? А любой химический источник тока это топливный элемент, только реагенты в твёрдом виде хранятся в основном. Хотя для возможности реакции всё равно их надо растворять в электролите, так что это не догма тоже.
477 588303
>>8302

>почему на порядок меньше энергии получится чем сжигать?


Мы говорим про реактивный движитель же.
Тут просто КПД тебе по зубам дает.
478 588304
image.png12 Кб, 552x127
479 588305
>>8304
Спизданул как господь.
480 588306
>>8302

>Берём водород, кислород, соединяем в топливном элементе, почему на порядок меньше энергии получится чем сжигать?


Так если у тебя водород и кислород уже есть - то проще сразу сжигать, чем в электричество гнать с непонятным КПД, и через тридевять преобразований с лишней массой преобразователей разгонять этим отдельно хранимое рабочее тело.
23192769176402433.pdf-8.jpg27 Кб, 720x540
481 588307
>>8303

>КПД тебе по зубам дает


Не на порядок. Меньше, конечно, зато гибкость применения выше - электричество вкуснее просто тепла.
482 588308
>>8307
Ты сжигая его и выбрасывая воду можешь при этом тепловую энергию в электричество преобразовывать. 50% КПД реактивной тяги + 1% электричества (грубо говоря)
Против 10% электрического КПД который ты хуй знает как задействуешь для тяги.

Нет, анончик, для тяги сжигать топливо это самое эффективное решение если мы говорим про э/м взаимодействие.
483 588309
>>8306
Зато появляется возможность весь выхлоп не выбрасывать, а сохранить и потом назад в компоненты перезарядить, как электричество появится.
484 588310
>>8308

>10% электрического КПД


Чё-то ты совсем батарейки принизил. По моему опыту, они вовсе даже не теряют 90% при разряде в тепло, и вообще при заряде только греются.
485 588311
>>8309
Это уже совсем тролльячья физика. Отброс выхлопа придаёт кинетическую энергию ракете. Собирая его обратно, ты остаёшься без ракеты.
486 588312
>>8310
Анон, когда батарея выделяет заложенную в нее энергию по максимуму - ты это видишь.
БОльшая часть энергии не расходутся при "полностью разряженной" батарее, ты по прежнему технически можешь выделить дохуя энергии из нее.
https://youtu.be/aRutyaxmdrs
Видео стронгли рилейтед.
И это наверное единственный вид электрического огня который можно и нужно тушить водой.
487 588313
>>8311
Не весь, а часть, конечно. За счёт высокого УИ электродвижков могу позволить, в принципе.
>>8312
Так и в ЖРД не все компоненты сгорают целиком. В водородниках даже подают на 20-30% больше водорода, чем по стехиометрии, чтобы УИ улучшить.
488 588314
>>8313

>Так и в ЖРД не все компоненты сгорают целиком. В водородниках даже подают на 20-30% больше водорода, чем по стехиометрии, чтобы УИ улучшить.


А я не утверждал, что ЖРД это 100% КПД.
Ты сейчас никак не опровергаешь мои слова, просто уточняешь.
Даже при таком неэффективном расходе оно выходит эффективно.
489 588327
поясните, кто в наса называет марсоходы? они троллят или это специально? сначала были спирит - дух и опотюнити - то есть возможность, потом кюрьёсити - любопытство, теперь персивиренс - упорство/назойливость. это что за путь горе-пикапера такой? возможность - любопытство - назойливость? что дальше будет, как следующий назовут может быть похоть или бесстыдство?
maxresdefault-2.jpg106 Кб, 1280x720
Выдуваем космические корабли 490 588334
Анончики, такая охуительная идея пришла в голову пару минут назад. Когда мы полетим таки к звёздам, нам потребуются OCHE большие корабли, ведь сверхсвета то нет, а топливо со жратвой нужно в очень больших количествах. Вопрос: как создавать корпуса таких кораблей? На Земле целиком не соберёшь, сделать отдельные части на Земле и собрать в космосе тоже не получается. Приходится полностью собирать в космосе. Но в невесомости прокатный станок не соберёшь, литейной формы охуительных размеров тоже нет. Идея такова: выдувать полые металлические корпуса (примерно как на пикриле) из расплавленного металла прямо в космосе. Вместо гигантского стеклодува используем сверхмощные насосы и инертные газы. Станок получается циклопических размеров, но довольно простой. Получившийся полый корпус затем обрабатывается, заполняется другими деталями, получается космический корабль (со стороны кстати на астероид похожий). Вопрос: в чём сложности такой идеи, и не хуйня ли это с самого начала?
491 588335
>>8334
У тебя ложный премис. Во-первых, можно собрать прокатный станок на орбите. Но это и не нужно, ты можешь запулять стальные листы и сваривать их на орбите.
Но самое обидное и реалистичное, что пока что никому это нахуй не всралось, никто вообще никак не собирается практически летать даже на марс.
492 588337
>>8334
Литьё сильно ограничено термодинамикой - с отводом тепла и с электричеством в космосе серьёзные проблемы.

Для орбитальной сборки больших герметичных объемов наиболее реалистичной считается роботизированная сборка из небольших шестиугольных плиток с последующей их сваркой. Она позволяет получать любые формы. Были предложения потестить схему на практике, эксперимент поставить сравнительно несложно и вполне реализуемо.

Но есть трансформируемые конструкции (aka надувнушки), которые в принципе ставят под сомнение целесообразность орбитальной сборки гермоконструкций. Потому что они надёжнее (всё проверяется ещё на Земле, а сборную конструкцию-то придётся испытывать в космических условиях, что намного сложнее и вообще хз как делать) и дают набор герметичных объемов, т.е. как раз то что и нужно вместо одного гигантского. Просто задумайся, что космический корабль, как и морской, должен всё равно быть поделен на герметичные отсеки, чтобы при пробоине весь воздух не выдуло сразу. Так что проще собирать из готовых трансформируемых модулей.
493 588338
>>8337

>Потому что они надёжнее (всё проверяется ещё на Земле, а сборную конструкцию-то придётся испытывать в космических условиях


Ещё и потому, что трансформируемый модуль более-менее целен, а у плиток огромное количество швов, каждый из которых это точка отказа. Собственно как и у любой сборной конструкции.
494 588364
>>8334
Блядь, бигелоу дал вам надувные модули из брезентовой хуйни, которые можно хоть до размеров дирижабля надувать.
495 588376
>>8335
Листов надо дофига, запульнуть не получится. Корабль же огромный.
И да, разве в невесомости можно создать проектный станок?

>>8337
1. Разве тепло не излучается в космос
2. Надувные конструкции вряд-ли будут выдерживать условия космоса тысячелетиями.
496 588378
>>8376

>Листов надо дофига, запульнуть не получится.


Тебе один хуй надо дофига чего-то запульнуть, пушо в космосе детали просто так не летают. В долгосрочной перспективе (если делать ну просто дохуя всего надо), проще запульнуть завод и майнить руду на астероидах. Но это такое отдаленное будущее, что писец, пока из сотен нужных для этого технологий у нас есть примерно ноль.
497 588379
>>8378
Именно, я изначально планировал добывать руду на астероидах, плавить их (солнечным светом например) и выдувать из расплавленного металла корпуса.
498 588380
>>8379
Ну, удачи, хуле.
image.png655 Кб, 640x425
499 588427
На мкс стоит винда? ЩИТО? Где вообще почитать что за ПО там стоит.
image.png20 Кб, 323x156
500 588432
>>8427
Не, там в основном дебиан повсюду.
Безымянный.png17 Кб, 800x450
501 588434
>>5265 (OP)
Все апполоны луну догоняли или шли наперерез?
503 588446
>>8435
Переходная орбита у них в ту же сторону была, с запада на восток, так что скорее догоняли. «Наперерез» случался только в самом конце, чтобы при свободном возврате сделать восьмёрку и вернуться обратно к Земле, двигаясь в том же самом восточном направлении.

Так и на старте меньше топлива тратится (т.к. вращение Земли добавляет скорости), и при торможении требуется меньше теплозащиты (т.к. атмосфера движется в попутном направлении).
504 588447
>>8446
Видимо, я не так вопрос понел. Разумеется на восток с добавлением скорости от земляшки, как иначе, они ж не из (((тех кто на запад ракеты запускает)))
505 588452
>>8447
Им Тора на восток запускать не велит?
506 588453
Я сейчас, наверное, совсем уж тупую хуйню спрошу, но у меня чёт ВРЁТИ и разрыв шаблона. Смотрел сегодня одного велопутешественника, он там едет по Перу через солончаки и кароч ориентироваться только по солнцу, ну он и спизданул что-то типа "Это же южное полушарие, тут солнце при движении с востока на запад отклоняется на север, а не как у нас на юг". Я что-то не так понял? Или он пизданул хуйню? Или это действительно так?
Таймкод- 10:30 https://www.youtube.com/watch?v=93RkIOrwyMk&t=1892s
507 588454
>>8452
У них на востоке злые хачи, на них ракеты ронять чревато.
508 588455
>>8453
Ну бля, Солнце над областью между тропиками, в районе экватора короч. Естественно, что от Перу это на севере.
509 588456
>>8453
Все верно, у нормальных людей солнце на южной половине неба, а там на северной.
510 588460
Как расширяется вселенная?
1. Если за пределами нет пространства, то ей ведь некуда расширяться. А так как расширяется пространство, то есть расширяется вселенная в саму себя? Или как это происходит в теории? То есть форма самой вселенной статична и сформирована?
2. Какой формы вселенная? Учитывая, что всё стремится к сфере - вселенная сферической формы?
511 588461
>>8460

> Как расширяется вселенная?


С ускорением. Пока это всё.
512 588474
>>8460
1. Само известное нам "пространство" находится внутри нашей вселенной. Раз расширяется — ну и пусть расширяется. Что находится за пределами — вселенной обсуждать смысла нет.
513 588504
Объясните, почему громадная кинетическая энергия, которой обладает космический аппарат на орбите, не превращается в тепловую при спуске?
Если, например, столкнуть 2 стальных шара, они нагреются. Кинетическая энергия преобразуется в тепловую.
Почему же не нагреваются тела космонавтов, приборы и всё остальное внутри корабля?
514 588505
>>8504
Еще как превращается, она вся уходит в нагрев воздуха и сжигание щита.
Потому, что они изолированы от плазмы, епт.
515 588508
Почему никто не передает энергию двигателю ракеты через лазер с земли
516 588509
>>8508
Хуйня получается, проще солярки с воздухом на борт взять и сжечь.
517 588510
>>8509
Почему проще?
518 588511
>>8510
Потому, что проще, заебал тупые вопросы задавать.
SS2168044.jpg87 Кб, 1199x434
519 588512
>>8504

>почему громадная кинетическая энергия, которой обладает космический аппарат на орбите, не превращается в тепловую при спуске?


Превращается. Спускаемый аппарат гонит ударную волну перед собой. В этой волне воздух почти моментально сжимается и от этого нагревается до состояния плазмы, разлагаясь на ионы. Типичная температура в наиболее горячих участках - 11-13 тыс градусов. Волна сама не касается аппарата, она бежит перед ним в нескольких сантиметрах. Но она всё равно передаёт большую часть своей энергии обратно аппарату, через тепловое излучение, т.к. раскалена до бешеной температуры.

>Почему же не нагреваются тела космонавтов, приборы и всё остальное внутри корабля?


Если речь о капсуле - у неё есть тепловой щит. Их бывает несколько разных типов.

Есть абляционный щит, он под действием высокой температуры испаряется с предсказуемой скоростью, постоянно испуская небольшое количество газа. Этот газ непрозрачен для инфракрасного (теплового) диапазона, и его непрерывный расходящийся в стороны поток образует тонкую подушку, отзеркаливающую излучение ударной волны наружу. Абляционные щиты используются для самых высокоскоростных аппаратов, входящих с межпланетных траекторий или прямо вниз, без планирования.

Аппараты с большой лобовой площадью относительно веса могут планировать, их можно не втыкать в атмосферу, а спускать под плавным углом, размазывая потерю энергии по более длинной траектории. Пиковый нагрев в таком случае меньше, и для отражения излучения достаточно плиток из специальной керамики, как у Шаттла, Бурана или X-37B.

Есть ещё раскрываемые замедлители (aka надувной щит, который на самом деле не всегда надувной, и вовсе не щит, а скорее парашют-замедлитель конической формы), у них огромная лобовая площадь и тормозят они очень хорошо, соответственно их можно спускать в планирующем режиме по совсем пологой траектории и размазывать потерю энергии по половине земного шара. Их можно делать из гибких материалов, например кевлара, который держит в районе 400 градусов. Таких пока было испытано полторы штуки, за пределами экспериментов и прототипов пока никто не применял - в основном не нравится низкая управляемость такого замедлителя, и низкая точность посадки.
SS2168044.jpg87 Кб, 1199x434
519 588512
>>8504

>почему громадная кинетическая энергия, которой обладает космический аппарат на орбите, не превращается в тепловую при спуске?


Превращается. Спускаемый аппарат гонит ударную волну перед собой. В этой волне воздух почти моментально сжимается и от этого нагревается до состояния плазмы, разлагаясь на ионы. Типичная температура в наиболее горячих участках - 11-13 тыс градусов. Волна сама не касается аппарата, она бежит перед ним в нескольких сантиметрах. Но она всё равно передаёт большую часть своей энергии обратно аппарату, через тепловое излучение, т.к. раскалена до бешеной температуры.

>Почему же не нагреваются тела космонавтов, приборы и всё остальное внутри корабля?


Если речь о капсуле - у неё есть тепловой щит. Их бывает несколько разных типов.

Есть абляционный щит, он под действием высокой температуры испаряется с предсказуемой скоростью, постоянно испуская небольшое количество газа. Этот газ непрозрачен для инфракрасного (теплового) диапазона, и его непрерывный расходящийся в стороны поток образует тонкую подушку, отзеркаливающую излучение ударной волны наружу. Абляционные щиты используются для самых высокоскоростных аппаратов, входящих с межпланетных траекторий или прямо вниз, без планирования.

Аппараты с большой лобовой площадью относительно веса могут планировать, их можно не втыкать в атмосферу, а спускать под плавным углом, размазывая потерю энергии по более длинной траектории. Пиковый нагрев в таком случае меньше, и для отражения излучения достаточно плиток из специальной керамики, как у Шаттла, Бурана или X-37B.

Есть ещё раскрываемые замедлители (aka надувной щит, который на самом деле не всегда надувной, и вовсе не щит, а скорее парашют-замедлитель конической формы), у них огромная лобовая площадь и тормозят они очень хорошо, соответственно их можно спускать в планирующем режиме по совсем пологой траектории и размазывать потерю энергии по половине земного шара. Их можно делать из гибких материалов, например кевлара, который держит в районе 400 градусов. Таких пока было испытано полторы штуки, за пределами экспериментов и прототипов пока никто не применял - в основном не нравится низкая управляемость такого замедлителя, и низкая точность посадки.
520 588513
>>8511
Ты мне зачем ответил, а? Чтобы повыпендриваться?
521 588514
>>8508
Передавать хотели, только не лазером, а микроволновкой. Было даже несколько концептов. На бумаге это интересная идея, но:
- проблема с фокусировкой луча на больших дистанциях
- облачность и вообще любая влажность/вода в воздухе поглощают энергию луча
- теплообменник должен быть на пузе, если качнётся или луч промажет, то всё внутри сгорит к херам
- теплообменник не идеален, и греется всё равно весь аппарат
- по мере подъёма угол аппарата по отношению к лучу меняется, соответственно меняется и КПД
- потенциальные пиздарики мимоспутникам, по которым может скользнуть луч такой мощности, направленный в небо
- регенеративное охлаждение затруднено, т.к. большая эффективность означает большую температуру рабочего тела, но и меньшее количество рабочего тела, возимое с собой. Т.е. более эффективный аппарат всегда будет больше нуждаться в холоде, но будет возить с собой меньше холода.
522 588517
>>8512
Это всё очень интересно, но не проясняет вопрос куда деваются 30 мегаджоулей каждого килограмма тушки огурца космонавта. Как она переходит в плазму-то, тем более если эта плазма даже не касается корпуса корабля?
523 588519
>>8517

>куда деваются 30 мегаджоулей каждого килограмма тушки огурца


Ты горишь где-то минут пять.
30Мдж/300 секунд = 100 киловатт
Вполне резонная цифра для ебаной плазмы.
524 588574
>>8517
Проясняет максимум детально.

>Спускаемый аппарат гонит ударную волну перед собой. В этой волне воздух почти моментально сжимается и от этого нагревается


Что именно тебе в этой фразе непонятно? Кинетическая энергия аппарата превращается в тепловую энергию волны сжатия.

>тем более если эта плазма даже не касается корпуса корабля


Это особенность ударной волны. Она возникает перед любым объектом, движущимся быстрее скорости звука в данной среде, и она тем дальше, чем больше разница их скоростей (число Маха). А аппарат при входе движется на ~22 махах.
525 588579
>>8517
Деваются в нагрев ударной волны через сжатие. Можно входить под большим углом, плюхаясь в атмосферу отвесно - тогда торможение будет более коротким и горячим. Можно входить полого, едва касаясь плотных слоёв вначале - тогда торможение будет медленней и холодней.
526 588580
>>8579

>Можно входить полого, едва касаясь плотных слоёв вначале - тогда торможение будет медленней и холодней.


Можно же случайно отскочить от атмосферы нахуй.
527 588582
>>8574

>Спускаемый аппарат гонит ударную волну


Расширю формулировку вопроса:
Как кинетическая энергия с мизинца левой ноги космонавта Ивана Говнова, сидящего в спускаемом аппарате, переносится на эту самую волну? По воздуху? По эфиру? По щучьему велению?
Почему, блять, эта энергия не превращается в тепловую "на месте", то есть там где она уже есть. Откуда энергия знает что ей нужно воздух сжимать и эту ёбаную ударную волну создавать?
528 588583
>>8582
Парень, ты... воопщи с ноший плонети? Космонавт давит на ложемент, ложемент на корпус, корпус на тепловой щит, который и сгорает.
Элементарно же. Хули космонавту воспламеняться, если реакция опоры действует весь путь?
529 588585
>>8582
Короче не я один во всей этой хуйне сомневаюсь -
The Apollo Space Program – A gigantic conspiracy?
Wolfgang G. Gasser
http://www.pandualism.com/d/apollo.html
А вы дальше скачите жопами на ударных волнах своих.
530 588586
>>8585
У тебя и самолеты не работают наверное, шизик?
Halley1986byESOgpo1386002-cc.jpg68 Кб, 800x1162
# OP 531 588588
Перекат, что ли.

>>588587 (OP)
>>588587 (OP)
>>588587 (OP)
532 588594
>>8585
Иди с миром, прозревший анончес!

Пошел на хуй, поехавший.
HeadofabarbarianAthens.jpg211 Кб, 515x660
533 588606
>>8582
>>8585
Блять, так бы сразу и сказал что ты шизоид, не осиливший 5 классов, не тратил бы на тебя время.
534 588607
>>8580
Можно, поэтому такое снижение должно быть прецизионным. А в принципе можно и отскоками тормозить, это ещё плавнее, и ещё больше требования к управляемости и точности.
Тред утонул или удален.
Это копия, сохраненная 29 января 2021 года.

Скачать тред: только с превью, с превью и прикрепленными файлами.
Второй вариант может долго скачиваться. Файлы будут только в живых или недавно утонувших тредах. Подробнее

Если вам полезен архив М.Двача, пожертвуйте на оплату сервера.
« /spc/В начало тредаВеб-версияНастройки
/a//b//mu//s//vg/Все доски